Mental Health Final

¡Supera tus tareas y exámenes ahora con Quizwiz!

Therapeutic Communication

Don't worry, everything will be ok, do not pass the problem, do not give advice, not give opinion, only provide neutral feedback), Silence, restating and reflecting, focus , clarification (put me an example), active listening, are good ways for doing therapeutic communication), open ended statement and questions are good, not do closed ended (yes or no) questions.

-Panic Attacks Nursing Instructions:

Take series of rapid shallow Breathing and concentrate in what happens in the body (to avoid hyperventilation and Respiratory Alkalosis)

A voluntarily hospitalized patient tells the nurse, "Get me the forms for discharge against medical advice (AMA) so I can leave the hospital now. "what is the nurse's best initial response? a- "I will get them for you, but let's talk about your decision to leave treatment." b- "I'll get the forms for you right now and bring them to you room." c- "I can't give you those forms without your health care provider's knowledge, you need to stay here until your provider see you" d- "Since you signed your consent for treatment, you may leave if you desire, it is ok."

a- "I will get them for you, but let's talk about your decision to leave treatment." -Voluntary (they may go AMA) and Involuntary Hospitalization, can not go AMA, Involuntary Hospitalization is only for patient that are in danger for him/herself or for another Person.

Parkinson's disease and Extrapyramidal side effects (EPS) of antipsychotics medications are the result of too little: a- Dopamine b- Norepinephrine c- GABA d- Serotonin

a- Dopamine -Neurotransmitters and applications: GABA (decreased in anxiety), Acetylcholine (decreased in Alzheimer and memory difficulty), Serotonin (decreased in Depression), Dopamine (increased in Schizophrenia and decreased in Parkinson disease), Norepinephrine (increased in anxiety).

A nurse uses Maslow's hierarchy of needs to plan care for a psychotic patient. which problem will receive priority? The patient: a- Who refuses to eat or bathe since 4 days ago b- Is reluctant to participate in unit social activities c- Reports feelings of alienation from family d- Needs to be taught about medication action and side effects.

a- Who refuses to eat or bathe since 4 days ago -Maslow Hierarchy of Needs, apply physiological (PHYSICAL NEEDS) needs first, apply ABC

Which statement made by a new client during an initial assessment interview should serve as the priority focus for the plan of care? Select one: a. " I am hearing voices that tell me to do very bad things." b. " I can not sleep well at night since 3 weeks ago" c. " You never know who will turn against you and it is horrible" d. "It seems like I always have bad luck."

a. " I am hearing voices that tell me to do very bad things."

A voluntarily hospitalized patient tells the nurse, "Get me the forms for discharge against medical advice (AMA) so I can leave the hospital now." What is the nurse's best initial response? a. "I will get them for you, but let's talk about your decision to leave treatment." b. "I'll get the forms for you right now and bring them to your room." c. "Since you signed your consent for treatment, you may leave if you desire, it is ok." d. "I can't give you those forms without your health care provider's knowledge, you need to stay here until your provider see you"

a. "I will get them for you, but let's talk about your decision to leave treatment."

A 26 month-old child displays many negative behaviors. The parent says, "My child refuses toilet training and shouts, 'No, no, no!' when given direction. What do you think is wrong?" Select the registered nurse's best reply. Select one: a. "This is normal for your child's age. The child is striving for independence." b. "The child needs more control. You have been weak" c. "There may be developmental problems. Most children are toilet trained by age 2 years and a hlaf." d. "Some undesirable attitudes are developing at this time. A child psychologist can help you develop a remedial plan."

a. "This is normal for your child's age. The child is striving for independence."

The nurse learns that a male client does not know the purpose of the medication ziprasidone (Geodon) that he is taking. How can the nurse best explain the purpose of this medication? a. "This medication will help you think more clearly." b. This is an antipsychotic medication to calm you down c. "An antipsychotic medication promote socialization " d. This medication helps people with schizophrenia

a. "This medication will help you think more clearly."

A nurse is caring for several clients who are attending community‐based mental health programs. Which of the following clients should the nurse plan to visit first? Select one: a. A client who says he is hearing a voice that tells him he is not worthy of living anymore b. A client who requests that her antipsychotic medication be changed due to some new adverse effects c. A client who tells the nurse he experienced manifestations of severe anxiety before and during a job interview d. A client who recently burned her arm by accident while using a hot iron at home

a. A client who says he is hearing a voice that tells him he is not worthy of living anymore

On review of the client's record, the nurse notes that the mental health admission was voluntary. Based on this information given, the nurse anticipates which client behavior? Select one: a. A willingness to participate in the planning of the care and treatment plan. b. No anger or aggressiveness directed toward others. c. Fearfulness regarding new treatment measures that provider ordered today d. An understanding all about the pathology and symptoms of the diagnosis.

a. A willingness to participate in the planning of the care and treatment plan.

A patient diagnosed with schizophrenia (SCPT) is acutely disturbed and violent. After several doses of ziprasidone (Geodon), the patient is calm. Two hours later the nurse sees the patient's head rotated to one side in a stiff position; the lower jaw is thrust forward, and the patient is drooling. Which problem is most likely? Select one: a. Acute dystonic reaction b. Waxy flexibility c. Tardive dyskinesia d. Stroke

a. Acute dystonic reaction

The registered nurse is caring for a client withdrawing from a Fentanyl Citrate addiction. The client receives a prescription for Clonidine 0.2 mg PO taken twice a day. Which action should the RN take? a. Advise to sit up slowly from a reclining position b. compare daily electrolyte levels prior to each morning dose c. administer a medication on an empty stomach d. Monitor for signs of bleeding and hemorrhage

a. Advise to sit up slowly from a reclining position

A client with depression is taking phenelzine (Nardil). The nurse advises the client to avoid consuming which foods while taking the medication Select one: a. Chocolate b. Cottage cheese c. Crackers. d. Oatmeal

a. Chocolate

Lithium is prescribed for a new patient. Which information from the patient's history indicates that monitoring serum concentrations of the drug will be especially challenging and critical? Select one: a. Congestive heart failure b. Psoriasis c. Dermatitis d. Epilepsy

a. Congestive heart failure

A client with schizophrenia returns to the clinic two weeks after receiving a prescription for haloperidol. To assess for Neuroleptic Malignant Syndrome (NMS) which information is most important for the registered nurse to obtain a. Current Vital signs b. Blood Sugar level c. 24 hours urinary output d. CBC

a. Current Vital signs

Parkinson's disease and Extrapyramidal side effects (EPS) of antipsychotics medications are the result of too little: Select one: a. Dopamine b. GABA c. Serotonin d. Norepinephrine

a. Dopamine

When reviewing the admission assessment, the Registered nurse notes that a client was admitted to the mental health unit with involuntarily status. Based on this type of admission, the registered nurse should provide which intervention for this client? Select one: a. Monitor closely for harm to a family member b. Monitor closely for severe anxiety and stress c. Monitor closely for using Methamphetamines d. Monitor closely for opioid overdose Feedback The correct answer is: Monitor closely for harm to a family member

a. Monitor closely for harm to a family member

When reviewing the admission assessment, the Registered nurse notes that a client was admitted to the mental health unit with involuntarily status. Based on this type of admission, the registered nurse should provide which intervention for this client? Select one: a. Monitor closely for harm to a neighbor b. Monitor closely for opioid consumption c. Monitor closely for using cocaine d. Monitor closely for severe anxiety

a. Monitor closely for harm to a neighbor

The nurse cares for a hospitalized adolescent diagnosed with major depressive disorder. The health care provider prescribes a low-dose antidepressant. In consideration of published warnings about use of antidepressant medications in younger patients, which action should the nurse employ? Select one: a. Monitor the adolescent closely for evidence of adverse effects, particularly suicidal thinking or behavior. b. Remind the health care provider about warnings associated with the use of antidepressants in children and adolescents c. Teach the adolescent about Black Box warnings associated with antidepressant medications d. Notify the facility's patient advocate about the new prescription.

a. Monitor the adolescent closely for evidence of adverse effects, particularly suicidal thinking or behavior.

Which medication should a nurse administer to provide immediate intervention for a psychotic patient whose aggressive behavior continues to escalate despite verbal intervention? Select one: a. Olanzapine (Zyprexa) b. Valproic acid (Depakene) c. Lithium (Eskalith) d. Trazodone (Desyrel)

a. Olanzapine (Zyprexa)

After change-of-shift report on the Alzheimer's disease/dementia unit, which patient will the nurse assess first? Select one: a. Patient who developed a NEW cough after eating breakfast b. Patient who has a stage II pressure ulcer on the coccyx c. Patient who is refusing to take the prescribed medications d. Patient who has not had a bowel movement for 5 days

a. Patient who developed a NEW cough after eating breakfast

Question text A female high school teacher, who was a child of alcoholic parents, seek counseling at a community clinic because of the depression after a student who was kill by a drunk driver. After several weeks of counseling, What client behavior is the best indicator that the client is coping well with the depression related to the student's death ? a. She becomes the school supporter/spoke person for the Student Against Drunk Driving (SADD) b. Describe alternatives to becoming depressed over the student's death c. Confront her parents about the hurt she felt as a child of d. Sign a safety contract with the nurse agreeing not to hurt herself or others

a. She becomes the school supporter/spoke person for the Student Against Drunk Driving (SADD)

A nurse is caring for a client who is experiencing a panic attack now . Which of the following actions should the registered nurse take? Select one: a. Stay with the client and remain quiet. b. Distract the client with a television show. c. Explain and demonstrate the client how to change his behavior. d. Discuss and explain new relaxation techniques.

a. Stay with the client and remain quiet.

The nurse would assess and take actions for neuroleptic malignant syndrome (NMS) if a client on haloperidol develops a: Select one: a. Temperature reading of 104.2° F and rigidity b. 20 mm Hg decrease in systolicblood pressure reading c. Temperature reading of 97.0° F and insomnia d. Respiratory rate of 22 respirations per minute

a. Temperature reading of 104.2° F and rigidity

A 69-year-old male patient who is hospitalized with pneumonia is disoriented and confused 3 days after admission. Which information indicates that the patient is experiencing delirium rather than dementia? Select one: a. The patient was oriented and alert when admitted. b. The patient's speech is slow. c. The patient has a history of increasing confusion over several years. d. The patient is oriented to person but disoriented to place and time.

a. The patient was oriented and alert when admitted.

A client with schizophrenia (SCPT) has been started on medication therapy with clozapine. The nurse should assess the results of which laboratory study to monitor for adverse effects from this medication? Select one: a. White blood cell count b. Liver function studies c. Blood glucose level d. Kidney function studies e. Platelet count

a. White blood cell count

A nurse receives this laboratory result for a patient diagnosed with bipolar disorder who has been taking Lithium carbonate PO for 2 months: lithium level 1.8 mEq/L. This result is: Select one: a. above therapeutic limits. b. likely to be inaccurate. c. below therapeutic limits. d. within therapeutic limits.

a. above therapeutic limits.

When administering a mental status examination to a patient with delirium, the nurse should do relaing with milieu therapy? Select one: a. choose a place without distracting stimuli. b. wait until the patient is well-rested. c. administer an anxiolytic medication. d. reorient the patient during the examination.

a. choose a place without distracting stimuli.

A client tells the nurse, "I wanted my health care provider to prescribe alprazolam (Xanax) for my anxiety disorder, but buspirone (Buspar) was prescribed instead. Why?" The registered nurse's reply should be based on the knowledge that buspirone: Select one: a. does not cause dependence and benzodiazepines cause dependence. b. has faster start of acting than diazepam. c. does not produce blood dyscrasias or low WBC count. d. can be administered as needed (PRN) only when you feel very anxious

a. does not cause dependence and benzodiazepines cause dependence.

A patient tells the nurse, "I wanted my health care provider to prescribe clonazepam (Klonopin) for my anxiety disorder, but buspirone (Buspar) was prescribed instead. Why?" The nurse's reply should be based on the knowledge that buspirone: Select one: a. does not cause dependence and benzodiazepines cause dependence. b. can be administered as needed when you feel very anxious. c. is faster acting than diazepam. d. does not produce blood dyscrasias or low WBC count.

a. does not cause dependence and benzodiazepines cause dependence.

A nurse can anticipate anticholinergic side effects will be likely when a patient is taking: Select one: a. haloperidol, typical antipsychotic medication b. buspirone, anxiolytic medication c. escitalopram, SSRI d. lithium, mood stabilizer

a. haloperidol, typical antipsychotic medication

A registered nurse is instructing a patient taking a drug that inhibits the action of monoamine oxidase enzyme (MAO) to avoid certain beverages, foods, and OTC medications because of the high risk of: Select one: a. hypertensive crisis. b. hypovolemic shock. c. orthostatic hypotension d. cardiac dysrhythmia and low stroke volume

a. hypertensive crisis.

A client diagnosed with major depressive disorder begins few days ago selective serotonin reuptake inhibitor (SSRI) antidepressant therapy. Priority information given to the patient and family should include a directive to: Select one: a. immediately report any increased suicidal thoughts. b. avoid exposure to bright sunlight. c. maintain a tyramine-free diet. d. restrict sodium intake to 0.5 g daily.

a. immediately report any increased suicidal thoughts.

Question text A patient experiences an episode of severe acute anxiety. Of these medications in the patient's medical record, which is most appropriate to administer as an as-needed (PRN) anxiolytic medication? Select one: a. lorazepam (Ativan) b. buspirone (Buspar) c. risperidone (Risperdal) d. fluoxetine (Prozac)

a. lorazepam (Ativan)

A health teaching plan for a patient taking lithium should include instructions to: Select one: a. maintain normal salt and fluids in the diet. b. avoid eating aged cheese, processed meats, and red wine. c. drink twice the usual daily amount of fluids. d. double the lithium dose if diarrhea or vomiting occurs.

a. maintain normal salt and fluids in the diet.

On the basis of current knowledge of neurotransmitter effects, a nurse anticipates that the treatment plan for a patient with memory difficulties may include medications designed to: Select one: a. prevent destruction of acetylcholine b. inhibit GABA production. c. increase dopamine sensitivity. d. increase serotonin sensitivity.

a. prevent destruction of acetylcholine

Goals and desired outcomes for an older adult patient experiencing delirium caused by fever and dehydration will focus on: Select one: a. returning to premorbid levels of function. b. identifying stressors negatively affecting self. c. demonstrating motor responses to noxious stimuli. d. exerting control over responses to perceptual distortions.

a. returning to premorbid levels of function.

A client is diagnosed with cancer and is told that surgery followed by chemotherapy will be necessary. The client states to the nurse, "I have read a lot about complementary therapies. Do you think I should try any?" The nurse should respond by making which appropriate statement? a. "Let's talk more about the different forms of complementary therapies." b. "If I were you I would try anything that I could if I had cancer." c. "No, because complementary therapies always interact with the chemotherapy." d. "I really don't know, you need to ask your health care provider about it."

a. "Let's talk more about the different forms of complementary therapies."

A school-age child tells the school nurse, "Other kids call me mean names and will not sit with me at lunch. Nobody likes me." Select the school nurse's most therapeutic response. a. "Tell me more about how you feel when it happens" b. Please tell me again everything from the beginning c. "Just ignore them and they will leave you alone, everyone has problems in the school" d. "Why do you want to sit with those children at lunch?"

a. "Tell me more about how you feel when it happens"

The registered nurse has written an outcome statement of "Client will feel less anxious by the end of session" for a client with generalized anxiety disorder (GAD). Which interventions should the licensed practical nurse use to assist this client in meeting this goal? Select all that apply. a. Administer anxiolytics medications if prescribed. b. Ensure the client feels he/she is in a safe environment. c. Refrain from speaking until the client's anxiety is decreased. d. Stay with the client. e. Give many detailed directions to the client during the session

a. Administer anxiolytics medications if prescribed. b. Ensure the client feels he/she is in a safe environment. d. Stay with the client. -Anxiety, stages and Medications (Benzodiazepines and buspirone, nursing considerations) , Mild anxiety is good, moderate or more not

A nurse wishes to teach alternative coping strategies to a patient who is experiencing severe anxiety. The nurse will first need to: a. Assess the patient's current anxiety level. b. Create outcomes and a teaching plan. c. Verify the patient's learning style. d. Teach the patient how to use defense mechanisms.

a. Assess the patient's current anxiety level.

A patient is diagnosed with social phobia. Which of the following would the healthcare identify as a characteristic of this disorder? a. Avoids interacting with strangers and speaking in public b. Fear of using public transportation c. Refuses to use a public restroom because of the virus and bacteria d. Avoids being in the presence of downs

a. Avoids interacting with strangers and speaking in public

A female client who's at high risk for suicide needs close supervision. To best ensure the client's safety, the Registered Nurse should: a. Check the client frequently at irregular intervals throughout the night b. Check the client frequently at regular intervals throughout the night c. Disregard decreased communication by the client because this is common in suicidal clients d. assure the client that the nurse will hold in confidence anything the client says

a. Check the client frequently at irregular intervals throughout the night -Suicide and Suicidal Precautions 1-(1:1 observation all the time), 2-doing frequent rounding at irregular intervals, the priority Nursing Diagnosis will be RISK FOR SUICIDE

The nurse provides care for a client with anorexia nervosa. The nurse knows which statements are true regarding anorexia nervosa? a. Clients diagnosed with anorexia nervosa often see themselves as over weight b. anorexia nervosa has the highest mortality of all mental disorders c. Clients diagnosed with anorexia nervosa often see themselves as emaciated and underweight d. Clients diagnosed with anorexia nervosa are self-indulgent e. Adolescent females are most affected.

a. Clients diagnosed with anorexia nervosa often see themselves as over weight b. anorexia nervosa has the highest mortality of all mental disorders e. Adolescent females are most affected.

A patient is diagnosed with agoraphobia. Which of the following would the healthcare identify as a characteristic of this disorder? a. Fears the use of public transportation b. Avoids interacting with strangers c. Refuses to use a public restroom d. Avoids being in the presence of clowns

a. Fears the use of public transportation

A nurse listens to a group of middle age adults in their 50s. One says, "I volunteer with Meals on Wheels, coach teen sports, and do church visitation." Another laughs and says, "I'm too busy taking care of myself to volunteer. I don't have time to help others." These comments contrast which development tasks according to Erikson Psychosocial Stages? a. Generativity versus Stagnation b. Integrity versus despair c. Intimacy versus isolation d. Industry versus Inferiority

a. Generativity versus Stagnation

The nurse documents the mental status of a client who has been hospitalized for several days by court order, The client states, "I don't need to be here" and tells the nurse that she believes the the television talks to her. The nurse should document these assessment findings in which section of the mental status exam? a. Insight and judgment b. Remote Memory (long-term memory) c. Mood and affect d. Level of concentration

a. Insight and judgment

Four patients in labor all request epidural analgesia to manage their pain at the same time. Which ethical principle is compromised when only one nurse anesthetist is on call? a. Justice b. Nonmaleficence c. Beneficence d. Fidelity

a. Justice

The nurse is preparing a client for the termination phase of the nurse-client relationship. The nurse prepares to implement which nursing task that is most appropriate for this phase? a. Making appropriate referrals b. Planning short-term and long-term goals c. Doing teaching and implementation d. Developing realistic solutions with the patient

a. Making appropriate referrals

While talking with a patient diagnosed with major depression, a nurse notices the patient is unable to maintain eye contact. The patient's chin lowers to the chest, while the patient looks at the floor. Which aspect of communication has the nurse assessed? a. Nonverbal communication b. A message filter c. A cultural barrier d. Social skills

a. Nonverbal communication

\While talking with a patient diagnosed with major depressive disorder, a nurse notices the patient is unable to maintain eye contact. The patient's chin lowers to the chest while the patient looks at the floor all the time. Which aspect of communication has the nurse assessed? a. Nonverbal communication b. Verbal communication c. A message filter d. Personality disorder of the patient

a. Nonverbal communication -Verbal and Non-Verbal Communication, the most important is Nonverbal (almost 90% of the communication happens using Nor verbal)

Which nursing intervention has priority as a patient diagnosed with anorexia nervosa begins to gain weight after initiated therapy? a. Observe for adverse effects of refeeding. b. Communicate empathy for the patient's feelings. c. Assess for depression and anxiety. d. Help the patient balance energy expenditure and caloric intake.

a. Observe for adverse effects of refeeding.

To plan effective care for patients with somatic system disorders, the nurse should understand that patients have difficulty giving up the symptoms because the symptoms: a. Provide relief from health anxiety. b. Can be voluntarily controlled. c. Are generally acute. d. Have a physiological basis.

a. Provide relief from health anxiety. -Somatic Symptom disorder, most common symptom is pain disorder, client has a lot of anxiety related to the symptoms, not physical reason for the symptom (example, pain), client usually refused or very resistant to receive mental health help

The nurse in the mental health unit recognizes which as being therapeutic communication techniques? Select all that apply. a. Restating b. Asking the client, "Why?" c. Providing acknowledgment and feedback d. Giving advice and approval or disapproval e. Providing false reassurance

a. Restating c. Providing acknowledgment and feedback

What is the most important goal of care for a client diagnosed with generalized anxiety disorder (GAD) who has been takin the benzodiazepine alprazolam long-term? The client will a. State the importance of not abruptly stopping the medication b. Every day describe a decrease in anxiety using a 1 to 10 anxiety scale c. Never experience dizziness, lightheadedness, or sedation d. Attend scheduled individual and group therapy sessions

a. State the importance of not abruptly stopping the medication

What is the nurse's priority assessment for a patient with borderline personality disorder? a. Suicidal or homicidal ideations b. Impulsive behaviors c. Only support systems d. Sleep pattern changes

a. Suicidal or homicidal ideations

Which assessment finding for a patient diagnosed with an eating disorder meets a criterion for hospitalization? a. Systolic blood pressure: 60 mm Hg b. Pulse rate: 52 beats/min c. Urine output: 90 ml/2 hr d. Serum potassium: 3.8 mEq/L

a. Systolic blood pressure: 60 mm Hg

After several therapeutic sessions with a patient who recently attempted suicide, which occurrence should cause the nurse to consider the possibility of countertransference? a. The nurse feels unusually and excessively happy when the patient's mood begins to lift. b. The patient states, "Talking to you feels like talking to my parents and uncle." c. The nurse develops a trusting relationship with the patient. d. The patient's reactions toward the nurse seem realistic and appropriate.

a. The nurse feels unusually and excessively happy when the patient's mood begins to lift.

A registered nurse reports to the interdisciplinary team that a patient with a personality disorder lies to other patients, verbally abuses a patient with dementia, and flatters the primary nurse. This patient is detached and superficial during counseling sessions. Which behavior most clearly warrants limit setting? a. Verbal abuse of another patient with dementia b. Detached superficially during counseling sessions c. Lying to other patients d. Flattering the nurse

a. Verbal abuse of another patient with dementia

A nurse uses Maslow's hierarchy of needs to plan care for a patient with mental illness. Which problem will receive priority? The patient: a. refuses to eat or bathe. b. reports feelings of alienation from family. c. is reluctant to participate in unit social activities. d. is unaware of medication action and side effects.

a. refuses to eat or bathe.

Which is the preferred treatment for change the false thoughts and ideas in patients with eating disorders? a. Expressive arts therapy b. Cognitive behavioral therapy c. Behavioral therapy only d. Always family therapy

b. Cognitive behavioral therapy

When developing a plan of care for a client to the psychiatric unit following aspiration of a caustic material related to a suicide attempt, which nursing diagnosis has the highest priority? a. Alteration in comfort b. Ineffective breathing patterns c. Ineffective coping mechanisms d. Risk for injury

b. Ineffective breathing patterns

A nurse sets limits for a patient diagnosed with a borderline personality disorder. The patient tells the nurse, "You used to be nice with me. I thought you were wonderful. Now I can see I was mistaken. You're terrible." This outburst can be assessed as: a. displacement b. splitting. c. rationalization d. transference

b. splitting -Defense Mechanisms (Denial, projection (the mother that does child abuse does not want the nurse touch her child), sublimation, altruism, passive-aggression), Reaction formation, splitting (Most frequent in Borderline Personality Disorder (BPD))

A depressed patient says to the nurse, "My life does not have any happiness in it anymore. I once enjoyed movies, but not anymore." How would the nurse document the complaint? Select one: a. Euphoria b. Vegetative c. Anhedonia d. Anergia

c. Anhedonia

A nurse observes a client who has OCD repeatedly applying, removing, and then reapplying makeup. The nurse identifies that repetitive behavior in a client who has OCD is due to which of the following underlying reasons? Select one: a. Narcissistic behavior b. Fear of rejection from staff c. Attempt to reduce anxiety d. Adverse effect of antidepressant medication

c. Attempt to reduce anxiety

What is a neurotransmitter? Select one: a. A gap between neurons b. A nerve cell c. Chemical messenger in the neurons d. The fatty covering of the axon

c. Chemical messenger in the neurons

The registered nurse instructs a patient taking a drug that inhibits the action of monoamine oxidase (MAO) to avoid certain foods and drugs because of the high risk of: Select one: a. cardiac dysrhythmia. b. hypotensive shock. c. Hypertensive crisis. d. cardiogenic shock.

c. Hypertensive crisis.

In a team meeting, a nurse says, "I am concerned if we are behaving ethically by using restraint to prevent one patient from self-mutilation while the care plan for another patient who has also self-mutilated issues calls for one-on-one supervision management ." Which ethical principle most clearly applies to this situation? Select one: a. Veracity b. Non maleficence c. Justice d. Beneficence

c. Justice

A patient has taken chlorpromazine (Thorazine) 200 mg/day orally for 3 years. The clinic nurse notes that the patient grimaces and constantly smacks both lips. The patient's neck and shoulders twist in a slow, snakelike motion. Which problem would the nurse suspect? Select one: a. Anticholinergic effects b. Akathisia c. Tardive dyskinesia d. Acute Dystonic Reaction

c. Tardive dyskinesia

A client is to begin valproic acid (Depakote) therapy for Bipolar disorder. The nurse will ensure that the client has completed what baseline lab work before the drug administration? a. Blood Sugar level b. kidney and pancreatic studies c. liver studies ( AST, ALT, Bilirubin, Alkaline Phosphatase) d. CBC

c. liver studies ( AST, ALT, Bilirubin, Alkaline Phosphatase)

A patient diagnosed with schizophrenia has taken Haldol , a first generation antipsychotic medication for 1 year. The delusions and auditory hallucinations are less intrusive but the patient continues to have lack of energy, flat affect, poverty of thought, and social isolation. The nurse expects a change to which medication? Select one: a. fluphenazine (Prolixin) b. clonazepam (Klonopin) c. quetiapine (Seroquel) d. chlorpromazine (Thorazine)

c. quetiapine (Seroquel)

A patient has symptoms of acute anxiety related to the death of a parent in an automobile accident 2 hours earlier. The nurse should anticipate administering a medication from which group? a. Mood stabilizers b. Atypical antipsychotics c. Benzodiazepines d. Tricyclic antidepressants

c. Benzodiazepines

A patient says, "I feel detached and weird all the time, like I'm looking at life through a cloudy window. Everything seems very unreal. These feelings really interfere with my work." Which term should the registered nurse use to document this complaint? a. Factitious disorder by proxy b. Dissociative identity disorder c. Depersonalization/Derealization disorder d. Conversion Disorder

c. Depersonalization/Derealization disorder

What is the most important goal for a client with Major Depression who has been receiving an antidepressant medication for two weeks? a. Ventilates feeling of sadness b. Eat three meals a day c. Does not attempt suicide d. Participates in group meeting

c. Does not attempt suicide

The most challenging nursing intervention for patients diagnosed with personality disorders, specially borderline personality disorder, who use manipulation to get their needs met is: a. Monitoring only dietary intake b. supporting multiple b behavioral change c. maintaining consistent limits d. using systematic desensitization

c. maintaining consistent limits

Low level of serotonin are linked to a- Generalized anxiety disorder b- Parkinson's disease c- Schizophrenia d- Depression

d- Depression

Too much dopamine is linked with a- Parkinson's disease b- Alzheimer disease c- Anxiety d- Schizophrenia (SCPT)

d- Schizophrenia (SCPT)

One specific type of antidepressant medication works by blocking the removal of neurotransmitters. Which of the following neurotransmitters is most likely the target of the antidepressant medication? a- GABA b- Dopamine c- Acetylcholine d- Serotonin

d- Serotonin

A client who abuses alcohol says to the nurse, I am glad I went in for treatment. Now my problems with alcohol are all behind me. Which response is best for the nurse to provide? Select one: a. You are wrong, you are likely to have a difficult time staying sober if you think that your problems with alcohol are behind you. b. Do you know what 'one day at a time' means for those who have problems with alcohol? c. Yes, in my opinion the treatment program you attended has an excellent success profile. d. Can you tell me more about what you mean when you say that your problems with alcohol are now behind you?

d. Can you tell me more about what you mean when you say that your problems with alcohol are now behind you?

Patients taking MAOIs have the tendency to experience hypertensive crisis especially during an interaction with other drugs such as epinephrine. Which of the following is a sign of hypertensive crisis? Select one: a. Palpitations b. Delay in ejaculation c. Orthostatic hypotension d. Diplopia

d. Diplopia

Too much dopamine is linked with Select one: a. Alzheimer disease b. Anxiety c. Parkinson's Disease d. Schizophrenia (SCPT)

d. Schizophrenia (SCPT)

A patient was started on escitalopram (Lexapro) 5 days ago and now says, "This medicine isn't working." The nurse's best intervention would be to: Select one: a. discuss with the health care provider the need to change the medication. b. assess the patient for symptom relief. c. reassure the patient that the medication will be effective soon. d. explain the time lag before antidepressants relieve symptoms.

d. explain the time lag before antidepressants relieve symptoms.

A 68-year-old patient is diagnosed with moderate dementia after multiple strokes. During assessment of the patient, the nurse would expect to find Select one: a. difficulty eating and swallowing. b. excessive nighttime sleepiness. c. fluctuating ability to perform simple tasks. d. loss of short-term and long-term memory, and wandering

d. loss of short-term and long-term memory, and wandering

A nurse should assess a patient taking a medication with anticholinergic properties for inhibited the function of the: Select one: a. sympathetic nervous system. b. pyramidal nervous system c. cerebellum and substantia nigra d. parasympathetic nervous system.

d. parasympathetic nervous system.

A patient's spouse field charges of battery. The patient says, "I'm sorry for what i did. I need psychiatric help." The patient has a long history of acting-out behaviors and several arrests. Which statement by the patient suggests an antisocial personality disorder? a. "I have a quick temper, but I hope I can keep it under control in the future." b. "I've done many stupid things in my life, but I've learned a lesson this time" c. I'm feeling bad about the way my behavior has hurt my family." d. "I hit because I'm tired of being nagged. My spouse deserves the beating."

d. "I hit because I'm tired of being nagged. My spouse deserves the beating."

On review of the client's record, the registered nurse notes that the mental health admission was voluntary. Based on this information, the nurse anticipates which client behavior? a. Ager and aggressiveness directed toward others b. Fearfulness regarding all treatment measures c. A full understanding of the pathology and symptoms of the diagnosis d. A willingness to participate in the planning of the care and treatment plan

d. A willingness to participate in the planning of the care and treatment plan

Over the past six months, a woman has cooked gourmet meals for her family but eats only tiny servings. She wears layered, loose clothing and now has lanugo. Her current weight is 97 pounds, a loss of 32 pounds. Which medical diagnosis is most likely? a. Bulimia nervosa b. Binge eating disorder c. Polyphagia d. Anorexia nervosa

d. Anorexia nervosa -Anorexia Nervosa, Hospitalization Criteria, HR less than 40 beats per minute, SBP (Systolic Blood Pressure) less than 70 mmHg, very low potassium

A nursing care plan for a patient with anorexia includes the intervention "monitor for complications of referring." Which system should a registered nurse closely monitor for dysfunction? a. Neurological b. Endocrine c. Musculoskeletal d. Cardiovascular

d. Cardiovascular

A nursing care plan for a patient with anorexia nervosa includes the intervention "monitor for complications of refeeding." Which system should a nurse closely monitor for dysfunction? a. Renal b. Endocrine c. Central nervous d. Cardiovascular

d. Cardiovascular

A young female client is admitted to the emergency room because she evening by her date. How should the registered nurse record the client's chief complaint in the medical record? a. Client has been sexually assaulted. b. Client claims that she was forced to participate in sexual intercourse. c. Client reported that she had sexual relations against her will. d. Client states, "my date raped me tonight."

d. Client states, "my date raped me tonight."

A mental nurse administers a medication that potentiates the action of gamma-aminobutyric acid (GABA). Which finding the nurse would be expected? a. Increased appetite b. Improve depression c. Fewer hallucinations and delusions d. Decreased Anxiety

d. Decreased Anxiety

What nursing assessment is the priority focus for a client with major depression: a. fluid and electrolyte balance. b. Mood and affect during the last 3 days c. Nutritional status d. Suicidal ideation

d. Suicidal ideation

A 21 year old client is brought to the ED with a suspected drug overdose. Which information is most important for the Registered Nurse to obtain for the family? a. Past history of depression b. The drug that was ingested c. Reason for the suicidal attempt d. The time since drug ingestion.

d. The time since drug ingestion.

The nurse learns that a male client does not know the purpose of the medication ziprasidone that he is taking. How can the nurse best explain the purpose of this medication? a. This medication helps people with schizophrenia b. An antipsychotic medication promotes socialization c. An antipsychotic medication calms you down d. This medication will help you think more clearly.

d. This medication will help you think more clearly.

A patient tells the nurse, "I wanted my health care provider to prescribe alprazolam (Xanax) for my anxiety disorder, but buspirone (Buspar) was prescribed instead. Why? I want my Xanax" The nurse's reply should be based on the knowledge that buspiroeusne: a. can be administered as needed (PRN for anxiety). b. it is only for short-term tratment c. is faster acting than diazepam. d. does not cause dependence and benzodiazepines cause dependence.

d. does not cause dependence and benzodiazepines cause dependence.

A student says, "Before taking a test, I feel a heightened sense of awareness and restlessness." The nurse can correctly assess the student's experience as: a. culturally influenced. b. displacement. c. trait anxiety. d. mild anxiety.

d. mild anxiety.

To assist a patient with a somatic system disorder, a nursing intervention of high priority is to: a. investigates only each physical symptom the patient is describing. b. helps the patient suppress feelings of anger. c. imply that somatic symptoms are not real d. shift the focus from somatic symptoms to feelings

d. shift the focus from somatic symptoms to feelings

Match the type of phobia with its corresponding characteristic. 1. Fear of open spaces: _______________ 2. Fear of interacting with others: _________ 3. Specific fear of objects or situations: __

1. Agoraphobia 2. Social phobia 3. Specific phobia

-Buspirone (Buspar)

nursing teaching, it is for anxiety long term treatment, take 3 to 6 weeks to achieve full anxiolytics effects, it is not PRN medication, best treatment for chronic anxiety (GAD) because it does not produce dependence)

Specific Phobias (to the snakes, spiders, plains, etc),

the treatment is Systematic desensibilisation

A psychiatric nurse assesses a client who commonly experiences anxiety. Which comment by this person indicates the possibility of obsessive-compulsive disorder (OCD) ? Select one: a. "I'm embarrassed to go out and speak in public." b. "My legs often feel weak and spastic." c. "I check where my car keys are eight times." d. "I keep reliving a car accident."

"I check where my car keys are eight times."

A nursing instructor is teaching about specific phobias. Which student statement should indicate that learning has occurred? A. "These clients do not recognize that their fear is excessive and rarely seek treatment." B. "These clients have a panic level of fear that is overwhelming and unreasonable." C. "These clients experience symptoms that mirror a cerebrovascular accident (CVA)." D. "These clients experience the symptoms of tachycardia, dysphagia, and diaphoresis."

B. "These clients have a panic level of fear that is overwhelming and unreasonable."

Group therapy is strongly encouraged, but not mandatory, on an inpatient psychiatric unit. The unit manager's policy is that clients can make a choice about whether or not to attend group therapy. Which ethical principle does the unit manager's policy preserve? A. Justice B. Autonomy C. Veracity D. Beneficence

B. Autonomy

Benzodiazepine antidote

Flumazenil

A patient should be considered for involuntary commitment for psychiatric care when he or she: Select one: a. threatens to harm self . b. sells and distributes illegal drugs in the community every weekend. c. is noncompliant with the very necessary treatment regimen. d. fraudulently files for bankruptcy last week.

a. threatens to harm self .

A nurse assesses a confused older adult. The nurse experiences sadness and reflects, "The patient is like one of my grandparents, so helpless and needy." What feelings does the nurse describe? Select one: a. Countertransference b. Defensive coping reaction c. Catastrophic reaction d. Transference

a. Countertransference

A nurse is caring for a client who smokes and has lung cancer. The client reports, "I'm coughing because I have that cold that everyone has been getting." The nurse should identify that the client is using which of the following defense mechanisms? Select one: a. Denial b. Suppression c. Sublimation d. Reaction formation

a. Denial

A nurse provides instructions to a client taking citalopram (Celexa) a selective serotonin reuptake inhibitors (SSRI) antidepressant. The nurse tells the client to take the medication: Select one: a. Early in the morning b. During lunch time c. At snack time in the afternoon d. At bedtime

a. Early in the morning

A new client is diagnosed with agoraphobia. Which of the following would the healthcare identify as a characteristic of this disorder? Select one: a. Fear of using public transportation b. Refuses to use a public restroom because of the virus, fungus, and bacteria c. Avoids interacting with strangers and speaking in public d. Avoids being in the presence of spiders

a. Fear of using public transportation

The nurse observes a client with PTSD, he is experiencing a dissociative disorder with quickly escalate. What intervention should the nurse implement first? a. Look for objects that can be used in a dangerous manner b. Encourage exploration of the feeling being expressed c. Call the physician d. Notify the nurse supervisor what is happening in the unit

a. Look for objects that can be used in a dangerous manner

A female client begins taking an atypical (second generation) antipsychotic medication. The nurse must provide informed consent and education about common medication side effects. Which client education will be most important? Select one: a. Maintain a balanced diet and adequate exercise, check your blood pressure and blood sugar b. Avoid taking the medication at night c. Avoid food rich in tyramine d. Be sure the diet is high in salt intake.

a. Maintain a balanced diet and adequate exercise, check your blood pressure and blood sugar

What is the priority nursing need for a patient diagnosed with late-stage dementia? Select one: a. Maintenance of nutrition and hydration b. Promotion of self-care activities c. Meaningful verbal communication d. Prevention of the patient from wandering

a. Maintenance of nutrition and hydration

According to Maslow's hierarchy of needs, which needs are most basic to a client's health maintenance plan? a. Physiological needs b. Steen and recognition c. Love and belonging d. Self -actualization

a. Physiological needs

The registered nurse in the mental health unit recognizes which as being good therapeutic communication techniques? Select all that apply. Select one or more: a. Reflecting b. Giving advice and approval or disapproval c. Change the subject d. Clarification e. Silence

a. Reflecting d. Clarification e. Silence

A patient whose schizophrenia has been refractory to treatment with other medications has been placed on clozapine (Clozaril). The priority discharge teaching should include: Select one: a. Report for weekly CBC blood test to detect agranulocytosis b. Maintain a tyramine-free daily diet. c. Always use sunblocking agents when out of doors. d. Report for weekly CBC blood test to detect only liver disease

a. Report for weekly CBC blood test to detect agranulocytosis

One specific type of antidepressant medication works by blocking the removal of neurotransmitters. Which of the following neurotransmitters is most likely the target of the antidepressant medication? Select one: a. Serotonin b. Acetylcholine c. GABA d. Dopamine

a. Serotonin

The nurse provides care for a client with anorexia nervosa. The nurse knows which statements are. true regarding anorexia nervosa? a. anorexia Nervosa has the highest mortality of all mental disorders b. Adolescent females are most affected c. Clients diagnosed with anorexia nervosa are self-indulgent d. Clients diagnosed with anorexia nervosa often see themselves as emaciated and underweight e. Clients diagnosed with anorexia nervosa often see themselves as overweight

a. anorexia Nervosa has the highest mortality of all mental disorders e. Clients diagnosed with anorexia nervosa often see themselves as overweight

A patient diagnosed with major depressive disorder begins selective serotonin reuptake inhibitor (SSRI) antidepressant therapy. Priority information given to the patient and family should include a directive to: Select one: a. immediately report increased suicidal thoughts. b. restrict sodium intake to 1 g daily. c. avoid exposure to bright sunlight. d. maintain a tyramine-free diet.

a. immediately report increased suicidal thoughts.

A patient cries as the nurse explores the patient's relationship with a decreased parent. The patient says, "I shouldn't be crying like this. It happened a long time ago." Which responses by the nurse will facilitate communication? Select all that apply. Select one or more a. "The loss of your parent should be very painful for you" b. "I can see that you feel sad about this situation." c. "Let's talk about something else, this subject is upsetting you, don't worry" d. "Don't be sad, everyone has to pass for something like this in the life" e. I felt very sad when my mother dies, it was horrible!

a. "The loss of your parent should be very painful for you" b. "I can see that you feel sad about this situation."

A woman is 5'8" tall, weighs 155 pounds, and wears a size 7.5 shoe. She says, "My feet are huge. I've asked tfour orthopedists to surgically reduce my feet, but they don't want." The patient tries to buy shoes to make her feet look smaller, and in social settings conceals both feet under a table or chair. Which health problem is likely? a. Body dysmorphic disorder b. Dissociative amnesia with fugue c. Depersonalization disorder d. Prominent pain disorder

a. Body dysmorphic disorder

a nurse plans teaching for a patient diagnosed with generalized disorder take clonazepam (klonopin). What information should be included? (Select all that apply) a. Use caution when driving. b. Take the medication on an empty stomach. c. Avoid taking the medication at night d. Maintain normal intake of shit e. Avoid using alcohol and other sedatives

a. Use caution when driving. e. Avoid using alcohol and other sedatives

Which physical characteristics are a registered nurse most likely to assess in a patient diagnosed with anorexia nervosa? a. Very low weight, langur, and amenorrhea b. Normal or high BMI, Russell Sign c. Rigidity, perfectionism, and lanugo d. Carefree flexibility and atrophic parodic glands

a. Very low weight, langur, and amenorrhea

A patient with paranoid schizophrenia believes evil spirits are being summoned by a local minister and verbally threatens to bomb a local church. The psychiatrist notifies the local minister. The psychiatrist has: a. demonstrated the duty to warn and protect b. avoided charges of malpractice c. released information without proper authorization d. violated the patient's confidentiality

a. demonstrated the duty to warn and protect

A psychiatric registered nurse best implements the ethical principle of autonomy when he or she: a. explores alternative solutions with a patient, who then make his/her own choice among the alternatives. b. stays with a patient who is demonstrating a severe level of anxiety c. suggests the two patients who have been fighting stay in their rooms d. intervenes when a self-mutilating patient attempts to harm him/her self

a. explores alternative solutions with a patient, who then make his/her own choice among the alternatives

A therapist recently convicted of multiple counts of Medicare fraud says, "Sure I overbilled. Why not? Everyone takes advantage of the government, so I did too." These statements show: a. lack of guilt feelings, antisocial personality disorder. b. suspiciousness. c. superficial remorse. d. shame.

a. lack of guilt feelings, antisocial personality disorder.

The mental health nurse is working with a client who shows signs of benzodiazepine withdrawal. The nurse suspects that the client has suddenly discontinued taking which of the following prescribed medications? a. lorazepam (Ativan) b. escitalopram (Lexapro) c. haloperidol (Haldol) d. fluoxetine (Prozac)

a. lorazepam (Ativan)

A supervisor assigns a worker a new project. The worker initially agrees but feels resentful. The next day, when asked about the project, the worker says, "I've been working on other things." When asked 4 hours later, the worker says, "Someone else was using the copy machine, so I couldn't finish it." The worker's behavior demonstrates: a. passive aggression. b. reaction formation c. altruism d. displacement

a. passive aggression.

A patient with a high level of motor activity runs from chair and cries, "They're coming! They're coming!" The patient does not follow the staff's directions or respond to verbal interventions. The initial Intervention of highest priority is to: a. provide for patient safety. b. encourage the clarification of feelings. c. decrease environmental stimuli. d. respect the patient's personal space.

a. provide for patient safety.

The nurse is developing a comprehensive care plan for a young woman with an eating disorder. The nurse refers this client to assertiveness skills classes. The nurse knows that this is an appropriate intervention because this client may have problems with a. self-identity and self-esteem. b. family boundary intrusions c. aggressive behaviors and lack of guilt d. focusing on reality.

a. self-identity and self-esteem.

An adult patient assaulted another patient and was restrained. One hour later, which statement by this restrained patient necessitates the nurse's immediate attention? Select one: a. "It was not my fault. The other patient started it." b. "My fingers are tingly." c. "You wait until I tell my lawyer." d. "I hate all of you!"

b. "My fingers are tingly."

A school-age child tells the school nurse, "Other kids call me mean names and will not sit with me at lunch. Nobody likes me." Select the nurse's most therapeutic response. Select one: a. "You should make friends with other children in the school, not with those kids" b. "Tell me more about how you feel when this happen." c. "Don't worry, just ignore them and they will leave you alone." d. "Why do you want to sit with those children at lunch?" Please explain me

b. "Tell me more about how you feel when this happen."

The nurse documents that a male client with paranoid schizophrenia is delusional. Which statement by the client confirms this assessment? a. "The voices are telling me to kill the next person I see." b. "The nurse at night is trying to poison me with pills." c. "The snakes on the wall are going to eat me." d. "The fire is burning my skin away right now."

b. "The nurse at night is trying to poison me with pills."

A 72-year-old female patient is brought to the clinic by the patient's spouse, who reports that she is unable to solve common problems around the house. To obtain information about the patient's current mental and short-term memory status, which question should the nurse ask the patient? Select one: a. "Are you sad?" b. "What did you eat for breakfast?" c. "How is your self-image?" d. "Where were you were born?"

b. "What did you eat for breakfast?"

-Tranylcypromine (MAOI)

patient needs further teaching if she/he said she/he will take the cold medicine if she/ha have a cold (Because many OTC medications have tyramine)

A patient who is experiencing moderate anxiety says, "I feel undone." An appropriate response for the registered nurse who need clarification would be: Select one: a. "Why do you feel like that? What would you like me to do to help you?" b. "I'm not sure I understand. Please give me an example... c. "Can you explain me everything again" d. "You must get your feelings under control before we can continue."

b. "I'm not sure I understand. Please give me an example...

A nurse is communicating with a client who was just admitted for treatment of a substance use disorder. Which of the following communication techniques should the nurse identify as a barrier to therapeutic communication? Select one: a. Reflecting b. Giving information c. Listening attentively d. Offering advice

d. Offering advice

A client is admitted to the in-patient unit and is being considered for electroconvulsive therapy (ECT). The client appears calm, but the family is hypervigilant and anxious. the client's mother begins to cry and states, "my child's brain will be destroyed. How can the doctor do this?" the nurse makes which therapeutic response? Select one: a. "In my opinion s you'd like to see the ECT room and speak to the staff" b. "it sounds as though you have some concerns about the ECT procedure. Let's sit down together and discuss any concerns you may have" c. "it sounds as though you need to speak to the psychiatrist." d. "your son has decided to have this treatment. you should be supportive of the decision"

b. "it sounds as though you have some concerns about the ECT procedure. Let's sit down together and discuss any concerns you may have"

The depletion of Acetylcholine is associated with: Select one: a. SCPT b. Alzheimer's disese c. Depression d. Acute Anxiety

b. Alzheimer's disese

When assessing a patient diagnosed with schizophrenia, which of the following will the healthcare provider identify as a negative symptom? Select one: a. Auditory Hallucinations b. Anhedonia c. Delusions d. Ilusions

b. Anhedonia

Group therapy is strongly encouraged, but not mandatory, on an inpatient psychiatric unit. The unit manager's policy is that clients can make a choice about whether or not to attend group therapy. Which ethical principle does the unit manager's policy preserve? Select one: a. Fidelity b. Autonomy c. Beneficence d. Justice

b. Autonomy

A male client in the mental health unit is guarded and vaguely answers the nurse's questions. He isolates to his room and sometimes opens the door to peek into the hall. Which problem can the nurse anticipate? a. Auditory hallucinations b. Delusions of persecution c. Visual hallucinations d. Increased motor activity

b. Delusions of persecution

A patient diagnosed with bipolar disorder (manic episode) commands other patients, "Get me a book. Take this stuff out of here," and other similar demands. The nurse wants to interrupt this behavior without entering into a power struggle. Select the best initial approach by the nurse. Select one: a. Humor: "How much are you paying servants these days?" b. Distraction: "Let's go to the dining room for a snack." c. Limit setting: "You must stop ordering other patients around." d. Honest feedback: "Your controlling behavior is annoying others."

b. Distraction: "Let's go to the dining room for a snack."

A young adult female client with panic disorder arrives in the Emergency Center with a 4-day history of chest pain that began when her boyfriend left her. Initial assessment reveals normal cardiopulmonary findings. Which information is most important for the nurse to obtain? a. Frequency of anxiety attacks. b. Drugs taken in last 7 days. c. Family history of suicide. d. Usual coping mechanisms.

b. Drugs taken in last 7 days.

holding a medication bottle labeled "lorazepam" What is the nurse's first priority action? Select one: a. Initiate vomiting b. Establish a patent airway c. Administer fluids d. Check only pupils and reflexes

b. Establish a patent airway

When developing a plan of care for a client to the psychiatric unit following aspiration of a caustic material related to a suicide attempt, which nursing diagnosis has the highest priority? a. Risk for injury. b. Ineffective breathing patterns. c. Ineffective coping mechanisms. d. Alteration in comfort.

b. Ineffective breathing patterns.

A patient receiving risperidone (Risperdal) reports severe muscle stiffness at 10:30 AM. By noon, the patient is diaphoretic, drooling, and has difficulty swallowing. By 4:00 PM, vital signs are body temperature, 102.8° F; pulse, 110 beats per minute; respirations, 26 breaths per minute; and blood pressure, 150/90 mm Hg. Select the nurse's best analysis and action. Select one: a. Cholestatic jaundice. Begin a high-protein, low fat diet. b. Neuroleptic malignant syndrome. Immediately notify the health care provider. c. Tardive dyskinesia. Withhold the next dose of medication. d. Agranulocytosis. Institute reverse isolation.

b. Neuroleptic malignant syndrome. Immediately notify the health care provider.

A patient receiving risperidone (Risperdal) reports severe muscle stiffness at 10:30 AM. By noon, the patient is diaphoretic, drooling, and has difficulty swallowing. By 4:00 PM, vital signs are body temperature, 103.5° F; pulse, 110 beats per minute; respiration, 27 breaths per minute; and blood pressure, 150/90 mmHg. Select the nurse's best analysis and action. Select one: a. Neuroleptic Malignant Syndrome, do nothing, it is a normal response b. Neuroleptic malignant syndrome. Immediately notify the health care provider. c. Tardive dyskinesia. Withhold the next dose of medication. d. Agranulocytosis. Institute reverse isolation.

b. Neuroleptic malignant syndrome. Immediately notify the health care provider.

A client tells the nurse that he wants to kill his boss, the registered nurse tells healthcare provider, healthcare provider tells his boss. What disciplinary action is needed? Select one: a. Both , the nurse and the healthcare provided did violation of confidentiality and privacy b. None. The action was appropriate due to duty to warm law c. File an incident report because violation of privacy has been done d. Inform the Board of Nursing that violation of patient's privacy

b. None. The action was appropriate due to duty to warm law

A patient diagnosed with schizophrenia has taken a first generation antipsychotic medication for a year. Hallucinations are less intrusive but the patient continues to have apathy, poverty of thought, and social isolation. The nurse expects a change to which medication? Select one: a. Haloperidol (Haldol) b. Olanzapine (Zyprexa), OR risperidone (Risperdal), OR quetiapine (Seroquel) c. Chlorpromazine (Thorazine) d. Diphenhydramine (Benadryl)

b. Olanzapine (Zyprexa), OR risperidone (Risperdal), OR quetiapine (Seroquel)

A patient diagnosed with Alzheimer Disease wanders at night. Which action should the nurse recommend for a family to use in the home to enhance safety? Select one: a. Obtain a bed with side rails b. Place locks at the tops of doors. c. Encourage frequent daytime napping to promote rest d. Place throw rugs on tile or wooden floors.

b. Place locks at the tops of doors.

The nurse manager on the psychiatric unit was explaining to the new staff the differences between typical and atypical antipsychotics. The nurse correctly states that atypical (Second generation ) antipsychotics: Select one: a. Produce fewer extrapyramidal effects (EPS) and target only negative symptoms of schizophrenia b. Produce fewer extrapyramidal effects (EPS) and target both positive and negative symptoms of schizophrenia c. Are risk free for neuroleptic malignant syndrome (NMS) , acute dystonia, and akathisia d. Produce fewer extrapyramidal effects (EPS) and target only positive symptoms of schizophrenia

b. Produce fewer extrapyramidal effects (EPS) and target both positive and negative symptoms of schizophrenia

A nurse administers a medication that potentiates the action of gamma-aminobutyric acid (GABA). Which finding the nurse would be expected? Select one: a. Improve depression b. Reduced anxiety c. Fewer sensory perceptual alterations d. Improve memory

b. Reduced anxiety

Which technique will best communicate to a patient that the nurse is interested in active listening? Select one: a. Ask a direct question, such as "Do you feel angry about that?" b. Restate a feeling or thought the patient has expressed c. Say, "I understand what you're saying." d. Make a positive judgment about the patient's problem.

b. Restate a feeling or thought the patient has expressed

When a hyperactive patient experiencing acute mania is hospitalized, what initial nursing intervention is a priority? Select one: a. Provide verbal instructions to the patient to remain calm. b. Setting limits on the patient's behavior as necessary. c. Allow the patient to act out his or her feelings. d. Restrain the patient to reduce hyperactivity and aggression.

b. Setting limits on the patient's behavior as necessary.

What is the most important goal of care for a client diagnosed with generalized anxiety disorder (GAD) who has been taking the benzodiazepine alprazolam (Xanax) for long-term? The client will Select one: a. Describe a decrease in anxiety using a 1 to 10 anxiety scale. b. State the importance of not abruptly stopping the medication. c. Not experience dizziness, lightheadedness, or sedation. d. Attend scheduled individual and group therapy sessions.

b. State the importance of not abruptly stopping the medication.

Which activity can the nurse delegate to unlicensed assistive personnel (UAP) who are working in a family practice clinic? Select one: a. Obtain patient histories regarding alcohol, tobacco, and other substance abuse. b. Take the vital signs for the patients in the clinic. c. Teach patients about the use of prescribed nicotine replacement products. d. Make referrals to community substance abuse treatment centers.

b. Take the vital signs for the patients in the clinic.

A patient diagnosed with major depressive disorder (MDD) is receiving imipramine (Tofranil) 200 mg every night at HS. Which assessment finding would prompt the nurse to collaborate with the health care provider regarding potentially hazardous side effects of this drug? Select one: a. Constipation b. Urinary retention c. Dry mouth d. Nasal congestion

b. Urinary retention

A nurse works with a patient diagnosed with schizophrenia regarding the importance of medication management. The patient repeatedly says, "I don't like taking pills." Which treatment strategy should the nurse discuss with the patient and health care provider? Select one: a. Addition of a benzodiazepine, such as lorazepam (Ativan) b. Use of long-acting antipsychotic injections c. Inpatient hospitalization because of the high risk for exacerbation of symptoms d. Adjunctive use of an antidepressant, such as amitriptyline (Elavil)

b. Use of long-acting antipsychotic injections

she was treaded for taking a deliberate overdose of her antidepressant medication, trazodone (desyrel). She says to the nurse, "My boyfriend broke up with me. We had been together for 6 years. I love him so much. I know I'll never get over him." Which is the best response by the nurse? Select one: a. Forget him. There are other fish in the sea b. You must be feeling very sad about your loss. c. Why do you think he broke up with you? d. You'll get over him in a few weeks, Theresa.

b. You must be feeling very sad about your loss.

The nurse is working with a client who shows signs of benzodiazepine withdrawal. The nurse suspects that the client has suddenly discontinued taking which of the following prescribed medications? Select one: a. sertraline (Zoloft) b. alprazolam (Xanax) c. fluoxetine (Prozac) d. haloperidol (Haldol)

b. alprazolam (Xanax)

A patient experiencing acute mania has disrobed in the hall three times in 2 hours. The nurse should: Select one: a. tell the patient that others feel embarrassed. b. arrange for one-on-one supervision. c. ask if the patient finds clothes bothersome. d. direct the patient to wear clothes at all times.

b. arrange for one-on-one supervision.

Alprazolam (Xanax) is prescribed for a patient experiencing acute anxiety. Most important teaching should include instructions to: Select one: a. adjust dose and frequency based on your anxiety level. b. avoid alcoholic beverages. c. report drowsiness. d. eat a tyramine-free diet.

b. avoid alcoholic beverages.

A priority nursing intervention for a patient diagnosed with major depressive disorder (MDD) is: Select one: a. offering opportunities for the patient to assume a leadership role in the therapeutic milieu. b. carefully, closely, and continuously observing the patient around the clock. c. distracting the patient from self-absorption. d. allowing the patient to spend long periods alone in self-reflection.

b. carefully, closely, and continuously observing the patient around the clock.

Which statement made by a new client during an initial assessment interview should serve as the priority focus for the plan of care? a. "You never know who will turn against you, it is horrible." b. "I am healing voices that tell me to do very bad things." c. "It seems like I always have bad luck." d. I can not sleep well at night since 3 weeks ago."

b. "I am healing voices that tell me to do very bad things."

A female victim of a sexual assault is being seen in the crisis center. The client states that she still feels "as though the rape just happened yesterday," even though it has been a few months since the incident. Which nursing response is appropriate? a. "It will take some times to get over these feelings about your rape." b. "Tell me more about the incident that causes you to feel like the rape just occurred." c. "What do you think that you can do to alleviate some of your fears about being raped again?" d. "You need to try and be realistic. The rape did not just occur."

b. "Tell me more about the incident that causes you to feel like the rape just occurred."

A 25 month-old displays many negative behaviors. The parent says, "My child refuses toilet training and shouts, "No, no, no!" When given direction. What do you think is wrong? Select the nurse's best reply. a. "Some undesirable altitudes are developing. A child psychologist can help you develop a remedial plan." b. "This is normal for you child's age. The child is striving for independence." c. "There may be developmental problems. Most children are toilet trained by age 2 years." d. "The child needs more control. Punish the child for disobedience and say, "No."

b. "This is normal for you child's age. The child is striving for independence."

The patient tells the nurse that she is afraid to speak up regarding her desire to end care for fear of upsetting her husband and children. Which principle in the nursing code of ethics ensures that the nurse will promote the patient's cause? a. Responsibility b. Advocacy c. Confidentiality d. Accountability

b. Advocacy

The registered nurse is preparing the orientation phase of the nurse-client relationship. The nurse prepares to implement which nursing task is the most appropriate and most important for this phase? a. Developing realistic solutions b. Built rapport and trust c. Making appropriate referrals d. Identifying expected outcomes

b. Built rapport and trust

The client with depression asks the nurse, "what are neurotransmitters? My doctor thinks my problem may lie with the neurotransmitters in my brain." What information should the nurse use to support an explanation of neurotransmitters? a. Web-like structures that provide connections among parts of the brain. b. Chemical messengers that cause brain cells to turn on or off. c. Areas of the brain that are responsible for controlling emotions d. Clumps of cells that alert the other brain cells to receive messages.

b. Chemical messengers that cause brain cells to turn on or off.

-When taking Benzodiazepines (Clonazepam, alprazolam, lorazepam),

patients need to AVOID: Alcohol, Antihistamines, Valerian Root, other sedatives)

A registered nurse assesses a confused older adult. The nurse experiences sadness and reflects, "The patient is like one of my grandparents, so helpless." What feelings does the nurse describe? a. Displacement b. Countertransference c. Repression d. Transference

b. Countertransference

A patient says, "I feel detached and weird all the time, like I'm looking at life through a cloudy window. Everything seems unreal. These feelings really interfere with my work and study." Which term should the nurse use to document this complaint? a. Dissociative identity disorder b. Depersonalization c. Factitious disorder d. Somatic Symptom Disorder

b. Depersonalization -Conversion disorder, loss of sensorial function, example: blindness, or motor function, example, paralysis, paraplegia, without physical reason or cause, usually after a traumatic event or a very severe stressful situation Depersonalization disorder Dissociative amnesia Dissociative amnesia with fugue Dissociative Identity Disorder (DID) (Multiple Personality Disorder) (See the questions in the PP)

A client on the mental health unit is becoming more agitated, shouting at the staff, and pacing in the hallway. When the PRN medication is offered, the client refuses the medication and defiantly sits on the floor in the middle of the unit hallway. What nursing intervention should the RN implement first? a. Administer medication to chemically restrain the patient. b. Take other clients in the area to the client lounge. c. Quietly approach the client with additional staff members. Transport of the client to the seclusion room.

b. Take other clients in the area to the client lounge.

A registered nurse is called to be home of a neighbor and finds and unconscious person still holding a medication bottle labeled "diazepam." What is the nurse's first action? a. Check pupils and reflexes b. Establish a patient airway c. Initiate vomiting and apply an enema d. Administer IV fluids

b. Establish a patient airway

A regsitered nurse puts a client who has a psychotic disorder in seclusion overnight because the unit is very short‐staffed, and the client frequently fights with other clients. the nurse's actions are an example of which of the following torts? Select one: a. Battery b. False imprisonment c.Invasion of privacy d.Assault

b. False imprisonment

In a Behavioral Health Unit team meeting, a registered nurse says, "I am concerned if we are behaving ethically by using restraint to prevent one patient from self-mutilation while the care plan for another patient who has also self-mutilated issues calls for one-on-one supervision." Which ethical principle most clearly applies to this situation? a. Autonomy b. Justice c. Non maleficence d. Veracity

b. Justice

The nurse observes a client with PTSD, he is experiencing a dissociative disorder with quickly escalate. What intervention should the nurse implement first? a. Call the physician b. Look for objects that can be used in dangerous manner c. Encourage exploration of the feeling being expressed d. Notify the nurse supervisor what is happening in the unit.

b. Look for objects that can be used in dangerous manner

Which activity can the nurse delegate to unlicensed assistive personnel (UAP) who are working in a family practice clinic? a. Obtain patient histories regarding alcohol, tobacco, and other substance abuse. b. Take the vital signs for the patients in the clinic. c. Make referrals to community substance abuse treatment centers d. Teach patients about the use of new prescribed nicotine replacement products for smoking cessation program.

b. Take the vital signs for the patients in the clinic.

Which nursing intervention has priority as a patient diagnosed with anorexia nervosa begins to gain weight after initiated therapy? a. Help the patient balance energy expenditure and caloric intake in every meal. b. Observe for adverse effects of refeeding and new onset of cardiac symptoms. c. Communicate empathy for the patient's feelings. d. Assess for depression and anxiety

b. Observe for adverse effects of refeeding and new onset of cardiac symptoms. -Anorexia Nervosa, Refeeding Syndrome, Cardiovascular Complications because acute overloud

The nurse recognizes which of these symptoms as characteristic of a panic attack? a. Increased respiratory rate, increased perceptual field, increased concentration ability. b. Palpitations, decreased perceptual field, diaphoresis, shortness of breath. c. Decreased blood pressure, chest pain, choking feeling. d. Increased blood pressure, bradycardia, shortness of breath.

b. Palpitations, decreased perceptual field, diaphoresis, shortness of breath. -PTSD, main symptom is Flashback, best nursing intervention is avoiding the triggers -Panic Attack,does not have triggers, because of that patient may developagoraphobia,

Which technique will best communicate to a patient that the registered nurse is interested in listening? a. Ask a direct question, such as, "Did you feel angry?" b. Restate a feeling or thought the patient has expressed. c. Giving advice and opinion about the patient's problem. d. Say, "I understand what you're saying now"

b. Restate a feeling or thought the patient has expressed.

The Health care provider prescribed lithium carbonate for a client diagnosed with Bipolar disorder. It is most important for the nurse to review which laboratory finding before start administering the medication? a. White Blood Cells Count b. Serum Creatinine c. Alkaline Phosphatase d. Blood Glucose

b. Serum Creatinine

Which principle should guide the nurse in determining the extent of silence to use during patient interview sessions? a. Patients withdraw if silences are frequent b. Silence can provide meaningful moments for reflection and it is therapeutic c. Silence helps patients know that what they said is well understood . d. Nurses are responsible for breaking silences.

b. Silence can provide meaningful moments for reflection and it is therapeutic

What nursing assessment is the priority focus for a client with major depression? a. Fluid and electrolyte balance b. Suicidal ideation c. Nutritional status d. Mood affect

b. Suicidal ideation

After several therapeutic sessions with a patient who recently attempted suicide, which occurrence should cause the nurse to consider the possibility of transference? a. The nurse develops a trusting relationship with the patient. b. The patient states, "Talking to you feels like talking to my parents." c. The nurse feels unusually and excessively happy when the patient's mood begins to lift. d. The patient's reactions toward the nurse seem realistic and appropriate for the nurse-patient relationship.

b. The patient states, "Talking to you feels like talking to my parents."

Susan, age 27, was admitted to the psychiatric unit from the medical intensive care unit when she was treated for taking deliberate overdose of her anxiolytic medication, clonazepam (Klonopin). She says to the nurse, "My boyfriend broke up with me. We had been together for 6 years. I love him so much. I know I'll never get over him." Which is the best response by the nurse? a. You'll get over him in a few weeks, Theresa. b. You must be feeling very sad about your loss. c. Why do you think he broke up with you? d. Forget him. The are other fish in the sea.

b. You must be feeling very sad about your loss.

A client is agitated and pacing in the hall near the nurses' station and swearing loudly. What response is best for the registered nurse to provide? a. Please go to your room to get control of yourself. b. You seem pretty upset. Tell me about it. c. Hey, what's going on? Be quiet. d. Others are being distracted. Please, quiet down and go to your room.

b. You seem pretty upset. Tell me about it.

A patient experiences an episode of severe anxiety. Of these medications in the patient's medical record, which is most appropriate to administer as an as-needed (PRN) anxiolytic medication? a. amitriptyline (Elavil) b. clonazepam (Klonopin) c. escitalopram (Lexapro) d. olanzapine (Zyprexa)

b. clonazepam (Klonopin)

A nurse should assess a patient taking a medication with anticholinergic properties for the following symptoms and signs: a. dry mouth, constipation, urinary retention, bradycardia b. dry mouth, constipation, urinary retention, tachycardia c. dry mouth, diarrhea, urinary retention, tachycardia d. dry mouth, constipation, urinary incontinence, tachycardia

b. dry mouth, constipation, urinary retention, tachycardia -Anticholinergic side effect in psychopharmacology S/SX (dry mouth, constipation, urinary retention, blurred vision, tachycardia), Nursing teaching: eat a rich fiber diet and plenty of fluid to decreased constipation), avoid Medication with anticholinergic effects in patients with Glaucoma, and BPH

Which scenario best demonstrates an individual: a. receives a bank notice there were insufficient funds in their account for a recent rent payment. b. is planning her/his weeding ceremony c. receives notification that their current employer is experiencing financial problems and some workers will be terminated d. loses a beloved family pet

b. is planning her/his weeding ceremony

Which scenario best demonstrates an example of EUSTRESS? An individual: a. receives notification that their current employer is experiencing financial problems lately and some workers will be terminated. b. prepares her/his wedding ceremony c. receives a bank notice there were insufficient funds in their account for a recent important payment. d. loses a beloved family pet.

b. prepares her/his wedding ceremony -Eustress (when you are planning a vacation, or a weeding)

What is the desirable outcome for the orientation phase of a nurse-patient relationship? The patient will demonstrate behaviors that indicate: a. resolution of feelings of dependance. b. rapport and trust with the nurse. c. self-responsibility and making progress with mood disorders. d. great sense of autonomy.

b. rapport and trust with the nurse. -Nursing-client relationship phases (Pre-orientation, Orientation, Working, and Termination) Orientation phase main task is Build the rapport and trust, Termination phase main task is doing proper referrals

A nurse set limits for a patient diagnosed with a borderline personality disorder. The patient tells the nurse, You used to care about me. I thought you were wonderful. Now I can see I was mistaken. Youre terrible. This outburst can be assessed as: a. denial. b. splitting. c. reaction formation. d. separation-individuation strategies.

b. splitting.

A woman who has just discovered that she is pregnant is in the clinic for her first obstetric visit. She asks the nurse, "How many drinks a day is safe for my baby?" The nurse's best response is: Select one: a. "It's okay to have up to two glasses of wine a day." b. "As long as you avoid getting drunk, you should be safe." c. "No amount of alcohol has been determined to be safe during pregnancy." d. "You should limit your drinking to twice or three times a week."

c. "No amount of alcohol has been determined to be safe during pregnancy."

The registered nurse is caring for a client withdrawing from a Fentanyl Citrate addiction. The client receives a prescription for Clonidine 0.2 mg PO taken twice a day. Which action should the RN take? a. Monitor for signs of bleeding and hemorrhage b. Administer a medication on an empty stomach c. Advise to sit up slowly from a reclining position d. Compare daily electrolyte levels prior to each morning dose.

c. Advise to sit up slowly from a reclining position

A patient diagnosed with bipolar disorder is in the maintenance phase of treatment. The patient asks, "Do I have to keep taking this lithium even though my mood is stable now?" Select the nurse's most appropriate response. Select one: a. "It's unusual that the health care provider has not already stopped your medication." b. "You will be able to stop the medication in approximately 1 year." c. "Taking the medication every day helps prevent relapses and recurrences." d. "Usually patients take this medication for approximately 6 months after initial diagnosis of Bipolar disorder."

c. "Taking the medication every day helps prevent relapses and recurrences."

An involuntary hospitalized patient tells the nurse, "Get me the forms for discharge against medical advice (AMA) so I can leave the hospital now." What is the nurse's best initial response? a- "I will get them for you, but let's talk about your decision to leave treatment." b- "I'll get the form for you right now and bring them to your room." c- "I can't give you those forms without your health care provider's knowledge, you need to stay here until your provider see you," d- "Since you signed your consent for treatment, you may leave if you desire, it is ok."

c- "I can't give you those forms without your health care provider's knowledge, you need to stay here until your provider see you,"

Cognitive therapy was provided for a patient who frequently said, "I'm dumb." Which statement by the patient indicate the therapy was effective? a- "I am disappointed in my lack of ability" b- "I always fail when I try new task and projects." c- "Sometimes I do things that are a little dumb." d- "I am disappointed in my lack of skills."

c- "Sometimes I do things that are a little dumb."

The depletion of acetylcholine is associated with: a- Depression b- SCPT c- Alzheimer's disease d- Acute anxiety

c- Alzheimer's disease

A patient comments, "I never know the right answer" and "My opinion is not important" using Erikson's theory, which psychosocial crisis did the patient have difficulty resolving? a- Initiative versus guilt b- Generativity versus self-absorption c- Autonomy versus shame and doubt d- Trust versus mistrust

c- Autonomy versus shame and doubt -Benzo Intoxication, priority Check and maintain open airway and respiration, Benzo are only for short term treatment, produce dependence, mechanism of action: increase GABA, never for long-term treatment , the antidote for Benzodiazepines is Flumazenil , NEVER COMBINE BENZO+ ALCOHOL, OR BENZO + OPIOID, OR BENZO + BARBITURIC OR ANY OTHER SEDATIVE. Ethical principles: Autonomy, Justice, Beneficence (do good), Veracity, Fidelity (keep your promises) -Erikson Stages (trust vs mistrust (infant task), autonomy vs shame and doubt, generativity vs stagnation and integrity vs despair)

According to Erickson's psychosocial stages of development, the stage in which a child needs to learn important academic skills and compare favorably with peers in school to achieve competence in the _____ stage. a- Initiative vs. guilt b- Trust vs. mistrust c- Industry vs. inferiority d- Identity vs. role confusion

c- Industry vs. inferiority

A cognitive strategy a nurse could use to assist a very dependent patient would be to help the patient: a- group therapy and role model ways to ask for help from others b- take prescribes medications c- examine thoughts about being autonomous and less dependent d- reveal dream content.

c- examine thoughts about being autonomous and less dependent

A cognitive therapy strategy a nurse could use to assist a very dependent patient would be to help the patient: a- take psychotropic prescribed medication b- reveal patient's dream content c- examine thoughts about being less dependent d- have delusion about being independent

c- examine thoughts about being less dependent Transference (from the patient onto the nurse) and Countertransference (from the nurse onto the patient), examples, review the Questions in PP -Cognitive Therapy (Cognitive Behavioral Therapy) -changes false thoughts and ideas

A schizophrenic client who is taking fluphenazine decanoate (Prolixin decanoate) is being discharged in the morning. A repeat dose of medication is scheduled for 20 days after discharge. The client tells the nurse about a planned vacation and will return in 18 days. Which statement by the client indicates to the nurse patient needs additional health teaching? Select one: a. "I will continue to take my benztropine mesylate (Cogentin) every day." b. "While I am on vacation, I will not eat or drink anything that contains alcohol." c. "I am going to have lots of fun at the beach and plenty of time in the sun." d. "I will notify the health care provider if I have a sore throat or flulike symptoms."

c. "I am going to have lots of fun at the beach and plenty of time in the sun."

An involuntarily hospitalized patient tells the nurse, "Get me the forms for discharge against medical advice (AMA) so I can leave the hospital now." What is the nurse's best initial response? Select one: a. "I will get them for you, but let's talk about your decision to leave treatment." b. "Since you signed your consent for treatment, you may leave if you desire, it is ok." c. "I can't give you those forms without your health care provider's knowledge, you need to stay here until your provider see you." d. "I'll get the forms for you right now and bring them to your room."

c. "I can't give you those forms without your health care provider's knowledge, you need to stay here until your provider see you."

A patient diagnosed with bipolar disorder is in the maintenance phase of treatment. The patient asks, "Do I have to keep taking this lithium even though my mood is stable now?" Select the nurse's most appropriate response. Select one: a. "Usually patients take this medication for approximately 6 months after discharge." b. "You will be able to stop the medication in approximately 1 month." c. "Taking the medication every day helps prevent relapses and recurrences." d. "It's unusual that the health care provider has not already stopped your medication."

c. "Taking the medication every day helps prevent relapses and recurrences."

A young adult female client is seen in the emergency department for a minor injury following a motor vehicle collision. She states she is very angry at the person who hit her car. What is the best nursing response? Select one: a. I understand your car was not seriously damaged, why are you so angry?." b. "You are lucky to be alive. Be grateful no one was killed." c. "You are upset that this incident has brought you here." d. Have you been in the emergency department before?"

c. "You are upset that this incident has brought you here."

A mental health nurse is giving instructions to a client receiving lithium carbonate medication twice a day. The nurse tells the client to do which of the following to prevent lithium toxicity: Select one: a. Always instruct the client to change positions slowly b. Limit fluid intake every day c. Avoid becoming dehydrated during exercise d. Restrict salt intake in the diet

c. Avoid becoming dehydrated during exercise

A young female client is admitted to the emergency room because she was raped that evening by her date. How should the registered nurse record the client's chief complaint in the medical record? Select one: a. Client reported that she had sexual relations against her will. b. Client has been sexually assaulted. c. Client states, "my date raped me tonight." d. Client claims that she was forced to participate in sexual intercourse.

c. Client states, "my date raped me tonight."

Low levels of serotonin are linked to Select one: a. Parkinson's disease b. Generalized Anxiety Disorder c. Depression d. Schizophrenia

c. Depression

A patient diagnosed with schizophrenia has taken fluphenazine (Prolixin) 5 mg orally twice daily for 3 weeks. The nurse now assesses a shuffling, propulsive gait; a masklike face; and drooling. Which term applies to these symptoms? Select one: a. Akathisia b. Hepatocellular effects c. Extrapyramidal symptoms (EPS) or Pseudoparkinson d. Neuroleptic malignant syndrome

c. Extrapyramidal symptoms (EPS) or Pseudoparkinson

A patient has anxiety, fear, and insomnia. The mental health nurse would suspect the presence of a decrease concentration of which neurotransmitter? Select one: a. Norepinephrine b. Histamine c. GABA d. Dopamine

c. GABA

A client who is being treated with lithium carbonate for bipolar disorder develops diarrhea, vomiting, and drowsiness. What action should the mental health nurse take? Select one: a. Record the symptoms as normal side effects and continue administration of the prescribed dosage. b. Notify the healthcare provider immediately and prepare for administration of an antidote. c. Hold the next dose of the medication and notify the healthcare provider of the symptoms prior to the next administration of the drug d. Hold the medication and refuse to administer additional amounts of the drug for the next five days

c. Hold the next dose of the medication and notify the healthcare provider of the symptoms prior to the next administration of the drug

The nurse on the evening shift received report that a client is scheduled for electroconvulsive treatment (ECT) in the morning. Which interventions should the nurse implement the evening before the scheduled ECT? a. Give client an enema at bedtime. b. Implement elopement precautions. c. Keep the client NPO after midnight. d. Hold all bedtime medications.

c. Keep the client NPO after midnight.

The nurse is caring for a client who is taking the mood stabilizer divalproex sodium (Depakote). Which laboratory finding is most important to include in this client's record? Select one: a. Creatinine and BUN b. CBC c. Liver function test results d. Sodium and potassium levels

c. Liver function test results

When reviewing the admission assessment, the Registered nurse notes that a client was admitted to the mental health unit with involuntarily status. Based on this type of admission, the registered nurse should provide which intervention for this client? Select one: a. Monitor closely for using cocaine b. Monitor closely for opioid consumption c. Monitor closely for harm to a neighbor d. Monitor closely for severe anxiety

c. Monitor closely for harm to a neighbor

A patient with a history of schizophrenia and who is taken risperido (Risperdal) every day is brought to the emergency department (ED). The patient has generalized muscle rigidity. Temperature, heart rate, and respiratory rate are elevated. These assessment findings are consistent with which of the following adverse effects of antipsychotic medications? Select one: a. Parkinsonism b. Serotonin Syndrome c. Neuroleptic malignant syndrome d. Tardive dyskinesia

c. Neuroleptic malignant syndrome

A patient diagnosed with schizophrenia has taken fluphenazine (Prolixin) 5 mg orally twice daily for 3 weeks. The nurse now assesses a shuffling, propulsive gait; a masklike face; and also patient is drooling. Which term applies to all these symptoms? Select one: a. Neuroleptic malignant syndrome b. Serotonin symptoms c. Parkinsonism or Extrapyramidal Symptoms (EPS) d. Akathisia

c. Parkinsonism or Extrapyramidal Symptoms (EPS)

Which hospitalized patient will the registered nurse assign to the room closest to the nurses' station? Select one: a. Patient with vascular dementia who is taking the emdications regularly. b. Patient with dementia who has an mild abnormal Mini-Mental State Examination c. Patient with new-onset confusion, restlessness, and irritability after surgery d. Patient with Alzheimer's disease who has long-term memory deficit

c. Patient with new-onset confusion, restlessness, and irritability after surgery

A client who is being treated with lithium carbonate for Bipolar Disorder type I begins to develop diarrhea, vomiting, and drowsiness. Which action should the registered nurse take? Select one: a. Notify the health care provider immediately and give 4 lithers of fluids b. Document the client's symptoms and continue with medication as prescribed c. Prior to giving the next dose, notify the health care provider of these symptoms and hold the next dose until new orders from Provider d. Hold the medication and refuse to administer additional doses for 3 days

c. Prior to giving the next dose, notify the health care provider of these symptoms and hold the next dose until new orders from Provider

A client who is being treated with lithium carbonate for manic depression begins to develop diarrhea, vomiting, and drowsiness. Which action should the nurse take? Select one: a. Document the symptoms and continue with medication as prescribed b. Hold the medication and refuse to administer additional doses for 3 days c. Prior to giving the next dose, notify the health care provider of these symptoms and hold the next dose until new orders from Provider d. Notify the health care provider immediately and force fluids

c. Prior to giving the next dose, notify the health care provider of these symptoms and hold the next dose until new orders from Provider

Which diet selection by a client who is depressed and taking the MAO inhibitor tranylcypromine sulfate (Parnate) indicates to the nurse that the client understands the dietary restrictions imposed by this medication regimen? Select one: a. Pepperoni and cheese pizza, tossed salad, and a soft drink. b. Liver and onions, broccoli, and ice coffee c. Roast beef, baked potato with butter, and orange juice. d. Hamburger, French fries, and chocolate milkshake.

c. Roast beef, baked potato with butter, and orange juice.

The nurse notes that a patient who has been receiving higfh doses of citalopram (Celexa) for symptoms of major depression begins to behave in a confused and elated manner with the presence of restlessness, rigidity, fever, muscle jerking, and diaphoresis. The nurse should assess these symptoms as probable: Select one: a. Acute dystonia b. Akathisia c. Serotonin syndrome d. Anticholinergic blockade

c. Serotonin syndrome

When reviewing the medical record of a patient diagnosed with Alzheimer disease (AD), the healthcare provider notes the patient is aphasic. Which behavior supports this finding? Select one: a. Unable to recognize objects b. Difficulty moving the extremities c. Unable to speak d. Difficulty swallowing

c. Unable to speak

Lorazepam (Ativan is prescribed for a patient experiencing acute anxiety. Most important teaching should include instructions to: Select one: a. report insomnia b. eat a tyramine-free diet. c. avoid taking other sedatives d. adjust dose and frequency based on your anxiety level

c. avoid taking other sedatives

A patient experiencing acute mania is dancing atop the pool table in the recreation room. The patient waves a cue in one hand and says, "I'll throw the pool balls if anyone comes near me." The nurse's first intervention is to: Select one: a. help the patient down from the table. b. assemble a show of force. c. clear the room of all other patients. d. tell the patient, "You need to be secluded."

c. clear the room of all other patients.

A client tells the nurse, "I wanted my health care provider to prescribe clonazepam (Klonopin) for my anxiety disorder, but buspirone (Buspar) was prescribed instead. Why?" The nurse's reply should be based on the knowledge that buspirone: Select one: a. can be administered as needed (PRN) only when you feel very anxious b. always produce blood dyscrasias or low WBC count. c. does not cause dependence and benzodiazepines cause dependence. d. has faster acting than diazepam.

c. does not cause dependence and benzodiazepines cause dependence.

A nurse wants to enhance the growth of a patient by showing positive regard. The action consistent with this wish is: Select one: a. staying with a tearful and very anxious patient b. making rounds every shift very carefully every 1 hour c. examining personal feelings about a new patient. d. administering medications as prescribed and PRN.

c. examining personal feelings about a new patient.

Systematic measurement of body weight, body mass index (BMI), waist circumference, and glucose levels would be most important for a patient beginning a new prescription for which medication? Select one: a. chlorpromazine (Thorazine) b. lithium carbonate c. olanzapine (Zyprexa) d. alprazolam (Xanax)

c. olanzapine (Zyprexa)

A voluntarily hospitalized patient tells the nurse, "Get me the forms for discharge. I want to leave now." Select the nurse's best response. a. "I will get the forms for you right now and bring them to your room." b. "Since you signed your consent for treatment, you may leave if you desire." c. "I will get them for you, but let's talk about your decision to leave treatment." d. "I cannot give you those forms without your health care provider's permission."

c. "I will get them for you, but let's talk about your decision to leave treatment."

A school-age child tells the school nurse, "Other kids call me mean names and will not sit with me at lunch. Nobody likes me." Select the nurse's most therapeutic response. Select one: a. "You should make friends with other children in the school, not with those kids." b. "Why you want to sit with those children at lunch?" c. "Tell me more about how you feel when it happen." d. "Tell me more about other things that are good, do not focus on this."

c. "Tell me more about how you feel when it happen."

The nurse documents that a male client with paranoid schizophrenia is delusional. Which statement by the client confirms this assessment? a. "The snakes on the wall are going to eat me." b. "The voices are telling me to kill the next person i see" c. "The nurse at night is trying to poison me with pills." d. "The fire is burning my skin away right now."

c. "The nurse at night is trying to poison me with pills."

The registered nurse has written an outcome statement of "Client will feel less the end of session" for a client with generalized anxiety disorder (GAD). Which interventions should the licensed practical nurse use to assist this client in meeting this goal? Select all that apply. a. Refrain from speaking until the client's anxiety is decreased. b. Give many detailed directions to the client at the same time during the session c. Administer anxiolytic medications as prescribed d. Stay with the client e. Ensure the client feels he/she is in a safe environment.

c. Administer anxiolytic medications as prescribed d. Stay with the client e. Ensure the client feels he/she is in a safe environment.

Over the past year, a woman has cooked gourmet meals for her family but eats only tiny servings. This person wears layered loose clothing. Her current weight is 95 pounds, a loss of 35 pounds. Which medical diagnosis is most likely? a. Binge eating b. Bulimia nervosa c. Anorexia nervosa d. Eating disorder not otherwise specified

c. Anorexia nervosa

A client with severe acute anxiety is admitted to the mental health unit. Which action should be implemented by the nurse, and not delegated to a unlicensed assistive personnel (UAP)? a. Check the patient's belongings b. Provide means for dietary selections c. Assess the current level of anxiety d. Obtain the vital signs of the patient

c. Assess the current level of anxiety

A 37-month-old child displays negative behavior, the child refuses toilet training, and often shouts, "No, no!" when given direction. The nurse's counseling with the parent should be based on the premise that the child is engaged in which of Erickson's psychosocial crises? a. Trust versus Mistrust b. Initiative versus Guilt c. Autonomy versus Shame and Doubt d. Industry versus Inferiority

c. Autonomy versus Shame and Doubt

A client postpartum depression receive prescription phenelzine (NARDIL). What information is most important to include in client teaching? a. Increase activity level to include a daily exercise routine. b. Maintain consistent intake of salt. c. Avoid processed meats, red wine, and swiss cheese. d. Contact health care provider immediately if muscle stiffness

c. Avoid processed meats, red wine, and swiss cheese.

A nurse plans teaching for a patient diagnosed with generalized anxiety disorder (GAD) who takes lorazepam (Ativan). What information should be included? (Select all that apply.) a. Maintain consistent intake of salt. b. Allow only tyramine-free foods in diet. c. Avoid using alcohol and other sedatives. d. Take the medication in the morning. e. Use caution when operating machinery.

c. Avoid using alcohol and other sedatives. e. Use caution when operating machinery.

Which of the following nursing interventions is MOST important when caring for a client who has just been placed in physical restraints? a. Monitor the client's needs for hydration and nutrition while restrained. b. Prepare PRN dose of psychotropic medication c. Check that the restraints have been applied correctly to avoid lack of circulation in the limbs d. Review hospital policy regarding duration of restraints.

c. Check that the restraints have been applied correctly to avoid lack of circulation in the limbs -Restrains, it is the last resort, check circulation is priority, is only for 4 hours RESTRAIN is NEVER PRN, put a patient in restrain without necessity and without a Provider order is Battery.

A client admitted to a mental health unit for treatment of psychotic behavior spends hours at the locked exit door shouting, "Let me out. There's nothing wrong with me. I don't belong here." What defense mechanism is the client implementing? a. Projection b. Suppression c. Denial d. Splitting

c. Denial

A veteran of the war in Afghanistan was diagnosed with post traumatic stress disorder (PTSD). The veteran says, "If there's a loud noise at night, I get under my bed because I think we're getting bombed." What type of experience has the veteran described? a. Illusion b. Nightmare c. Flashback d. Delusion

c. Flashback

A client being admitted to the eating disorders unit has fine, downy hair covering the body. The patient weighs 74 pounds; height is 5 feet, 5inches, and has amenorrhea. The patient is quiet and says only, "I won't eat anything until I look thin." What is the priority initial nursing diagnosis? a. Anxiety, related to fear of weight gain b. Disturbed body image, related to weight loss c. Imbalanced nutrition: less than body requirements, related to self-starvation d. Imbalanced nutrition: more than body requirements, related to binge eating

c. Imbalanced nutrition: less than body requirements, related to self-starvation

An adult female patient with bipolar disorder is seen in the outpatient psychiatric unit and tell the nurse that she is thinking of harming her sister. Which action is most important for the RN to take? a. Notify the threat to the healthcare provider (HCP) b. Document the threat in the Medical Record c. Inform the sister of the client threat d. Report the threat to the Healthcare team.

c. Inform the sister of the client threat

A client who was recently diagnosed with Anorexia Nervosa collapses at a community clinic. The client begins to demonstrate cloudy consciousness, stupor and slurred speech. The nurse obtain the blood pressure 88/50 mmHg, pulse of 116, BS 50 mg/dl. Which intervention is most important for the RN to implement? a. Reinforce the need to continue the outpatient clinic therapy b. Encourage the client to eat low-carbohydrate and high-protein meals. c. Position client with head flat and feet elevated. d. Suggest obtaining a medical alert bracelet to be always worn.

c. Position client with head flat and feet elevated.

Too much dopamine is linked with a. PTSD b. Depression c. Schizophrenia (SCPT), Psychosis d. Parkinson's Disease

c. Schizophrenia (SCPT), Psychosis

Which assessment finding for a patient diagnosed with an eating disorder meets a criterion for hospitalization? a. Serum potassium: 3.8 mEq/L b. Urine output: 70 ml/2 hours c. Systolic blood pressure: 62 mm Hg d. Pulse rate: 56 beats/min

c. Systolic blood pressure: 62 mm Hg -Eating Disorder, Anorexia Nervosa and Bulimia Nervosa, monitoring and avoid refeeding syndrome, the patient need to be feeding slow, should not gain more than 2 pounds a week, the most common complication in refeeding Syndrome are cardiovascular complications

The Healthcare provider prescribed clozapine for a client diagnosed with paranoid schizophrenia. It is most important for the nurse to review which laboratory finding before start administering the medications? a. Fasting Blood sugar b. Serum Creatinine c. White Blood Count (WBC) d. Direct Bilirubin e. Alkaline Phosphatase

c. White Blood Count (WBC)

A community mental health nurse has worked with a patient for 2 years but now she is moving out of the city and terminates the relationship. A new nurse who begins work with this patient will: a. return to the emotional catharsis phase. b. resume the working relationship as soon as possible. c. begin at the orientation phase. d. enter into a social relationship.

c. begin at the orientation phase.

A patient tells the registered nurse, "I wanted my health care provider to prescribe clonazepam (Klonopin) for my anxiety disorder, but buspirone (Buspar) was prescribed instead today. Why?" The registered nurse's reply should be based on the knowledge that buspirone: a. does not produce liver or kidney disorders b. can be administered as needed for anxiety when the client needs the medication c. does not cause dependance and benzodiazepines cause dependence d. is always faster acting than clonazepam

c. does not cause dependance and benzodiazepines cause dependence

A single parent is experiencing feelings of inadequacy related to work and family since one teenaged child ran away 2 weeks ago. The parent seeks the help of a therapist specializing in cognitive therapy. The psychotherapist who uses cognitive therapy will treat the patient by: a. focusing only on unconscious mental processes b. always discussing Freud ego stages c. helping the patient how to identify and change faulty thinking d. negatively reinforcing an undesirable behavior every day

c. helping the patient how to identify and change faulty thinking

A therapist recently convicted of multiple counts of Medicare fraud says, Sure I overbilled. Why not? Everyone takes advantage of the government, so I did too. These statements show: a. shame. b. decreased empathy c. lack of remorse or guilt d. suspiciousness

c. lack of remorse or guilt

The most challenging nursing intervention for patients diagnosed with personality disorders, especially borderline personality disorder, who use manipulation to get their needs met is: a. supporting behavioral change. b. administering prescribed medications c. maintaining consistent limits. d. using aversive and systematic desensitization therapy.

c. maintaining consistent limits. -Personality Disorders (Custer b, Borderline and Antisocial), core symptoms and Nursing Interventions : Setting and Maintaining Consistent Limits), do frequent staff team meeting for maintain the consistent limits and to avoid manipulation)

A patient should be considered for involuntary commitment for psychiatric care: a. fraudulently files for bankruptcy 3 times b. sells and distributes illegal drugs in the community c. threatens to harm others d. is noncompliant with the very important treatment regimen

c. threatens to harm others

Which describes the primary focus of milieu therapy? Select one: a. A cognitive approach to changing manipulative behavior b. A behavioral approach to changing behavior c. A form of behavior modification therapy used in psychotic patients d. A living, learning, or working environment

d. A living, learning, or working environment

A nurse is called to the home of a neighbor and finds an unconscious person still holding a medication bottle labeled "lorazepam." What is the nurse's first action? Select one: a. Initiate vomiting b. Check pupils c. Test reflexes d. Establish a patent airway

d. Establish a patent airway

A patient is prescribed haloperidol (Haldol) for the management of schizophrenia. Before administering the medication to the patient, the healthcare provider observes facial grimacing and tongue thrusting. Which of the following interventions should the healthcare provider perform first? Select one: a. The patient is exhibiting a prodromal symptom of seizures. b. Send a blood sample to the lab to measure the haloperidol level c. Administer benztropine (Congentin) d. Hold the medication and continue to assess the patient

d. Hold the medication and continue to assess the patient

A patient diagnosed with major depressive disorder has lost 15 pounds in one month. The patient has chronic low self-esteem and a plan for suicide. The patient has taken an antidepressant medication for 1 week. Which nursing intervention is most directly related to this priority: "Patient will refrain from gestures and attempts to harm self"? Select one: a. Assess the patient for therapeutic effects of antidepressant medication taken b. Assist the patient to identify three personal strengths and weakness c. Frequently offer snacks and water d. Implement suicide precautions immediately

d. Implement suicide precautions immediately

A client with a history of four previous suicide attempts during the last 2 years has been taking citalopram (Celexa) for 1 month. The client suddenly presents with a bright affect, rates mood at 9/10, and is much more communicative. Which action should be the nurse's priority at this time? Select one: a. Give the client off-unit privileges as positive reinforcement. b. Encourage the client to share mood improvement in group. c. Request that the psychiatrist reevaluate the current medication protocol d. Increase frequency of client observation and maintain closely observation .

d. Increase frequency of client observation and maintain closely observation .

When preparing a teaching plan for a client who is to be discharged with a prescription for buspirone (Buspar), it is most important for the nurse to include which instruction? Select one: a. Avoid eating aged cheese and chicken liver. b. Eat foods high in fiber such as whole grain breads. c. Eat a normal intake of salt every day d. It may take 3 to 4 weeks to achieve therapeutic effects

d. It may take 3 to 4 weeks to achieve therapeutic effects

When preparing a teaching plan for a client who is to be discharged with a prescription for lithium carbonate (Lithonate), it is most important for the nurse to include which instruction? Select one: a. Eat foods high in fiber such as whole grain breads. b. Avoid eating aged cheese and chicken liver. c. It may take 3 to 4 months to achieve therapeutic effects. d. Keep your dietary salt intake consistent.

d. Keep your dietary salt intake consistent.

The nurse manager on the psychiatric unit was explaining to the new staff the differences between typical and atypical antipsychotics. The nurse correctly states that atypical antipsychotics: Select one: a. Remain in the system longer b. Act more quickly to reduce delusions and auditory hallucinations c. less risk for neuroleptic malignant syndrome (NMS) d. Produce fewer and less severe extrapyramidal effects and target both positive and negative

d. Produce fewer and less severe extrapyramidal effects and target both positive and negative

A mental health nurse administers a medication that potentiates the action of gamma-aminobutyric acid (GABA). Which finding would be expected? Select one: a. More organized thinking process b. Fewer perceptual alterations c. Improved patient's memory d. Reduced level of anxiety

d. Reduced level of anxiety

A nurse in an acute mental health facility is communicating with a client. the client states, "I can't sleep. I stay up all night." the nurse responds, "You are having difficulty sleeping?" Which of the following therapeutic communication techniques is the nurse demonstrating? a. Silence b. Offering general leads c. Focusing d. Restating

d. Restating

A patient has taken trifluoperazine (Stelazine) 30 mg/day orally for 3 years. The clinic nurse notes that the patient grimaces and constantly smacks both lips. The patient's neck and shoulders twist in a slow, snakelike motion. Which problem would the nurse suspect? Select one: a. Agranulocytosis b. Anticholinergic effects c. Tourette syndrome d. Tardive dyskinesia

d. Tardive dyskinesia

A client taking lithium carbonate (Lithobid) started complaining of nausea, vomiting, diarrhea, drowsiness, muscle weakness, tremor, blurred vision and ringing in the ears. The lithium level is 2.2 mEq/L. The nurse interprets this value as: Select one: a. Below normal level b. At therapeutic levels c. Within normal levels d. Toxic level

d. Toxic level

A patient diagnosed with major depressive disorder (MDD) is receiving imipramine (Tofranil) 200 mg PO every night at HS. Which assessment finding would prompt the nurse to collaborate with the health care provider regarding the most potentially hazardous side effects of this drug? Select one: a. Dry mouth b. Nasal congestion at night c. No bowel movement for 2 days d. Urinary retention

d. Urinary retention

A registered nurse works with a patient diagnosed with schizophrenia regarding the importance of medication management. The patient repeatedly says, "I don't like taking all those pills." Which treatment strategy should the nurse discuss with the patient and health care provider? Select one: a. Inpatient hospitalization because of the high risk for exacerbation of psychotic symptoms b. Adjunctive use of an antidepressant, such as amitriptyline (Elavil) to improve patient's mood c. Addition of a benzodiazepine, such as lorazepam (Ativan) to decrease anxiety d. Use of long-acting antipsychotic intramuscular like Risperdal Consta

d. Use of long-acting antipsychotic intramuscular like Risperdal Consta

The nurse is concerned about a postoperative patient's risk for injury during an episode of delirium. The most appropriate action by the nurse is to Select one: a. instruct family members to remain with the patient and prevent injury. b. ask the health care provider to order an antipsychotic drug. c. secure the patient in bed using a soft chest restraint. d. assign unlicensed assistive personnel (UAP) to stay with the patient and offer reorientation.

d. assign unlicensed assistive personnel (UAP) to stay with the patient and offer reorientation.

A patient was started on fluoxetine (Prozac) 5 days ago and now says, "This medicine isn't working." The nurse's best intervention would be to: Select one: a. discuss with the health care provider the need to change the medication. b. assess the patient for symptom relief. c. reassure the patient that the medication will be effective tomorrow d. explain the antidepressants take from 2 to 4 weeks to relieve symptoms.

d. explain the antidepressants take from 2 to 4 weeks to relieve symptoms.

A Registered Nurse is developing a teaching plan for a patient who starts taking lithium carbonate PO twice a day, the teaching plan should include instructions related to: Select one: a. avoid eating cheese , processed meats, and tea. b. drink tree times the daily amount of fluids in order to stay overhydrated c. double the lithium dose at night if nausea, diarrhea or vomiting occurs. d. maintain normal intake of salt and fluids in the patient's diet.

d. maintain normal intake of salt and fluids in the patient's diet.

A nurse receives this laboratory result for a patient diagnosed with bipolar disorder: lithium level 0.9 mEq/L. This result is: Select one: a. above therapeutic limits. b. likely to be inaccurate. c. below therapeutic limits. d. within therapeutic limits.

d. within therapeutic limits.

An older adult patient who has been taking alprazolam (xanax) calls the clinic asking for a refill of the prescription 1 month before the alprazolam should need to be refilled. Which response by the nurse is best? a. "Do you have any muscle cramps and tremors if you don't take the medication frequently?" b. "Don't worry, I will ask the doctor to prescribe a few more pills, but you will not be able to get any more for another month." c. "The prescription cannot be refilled for another month. What happened to your pills?" d. "I am concerned that you may be overusing the alprazolam. Let's make an appointment for you to see the doctor."

d. "I am concerned that you may be overusing the alprazolam. Let's make an appointment for you to see the doctor."

A patient who is experiencing moderate anxiety says, "I feel undone." An appropriate responses for the registered nurse who need clarification would be: a. "You must get your feelings under control before we can continue." b. "Why do you feel like that? What would you like me to do to help you?" c. "Can you explain me everything again" d. "I'm not sure I understand. Please give me an example..."

d. "I'm not sure I understand. Please give me an example..."

Which best describes a characteristic of bulimia nervosa? a. Following a very strict diet and exercise program b. Always avoiding social gatherings and family meals c. Restricting caloric intake all the time d. Bingeing on unhealthy food and purging after, to induce vomiting

d. Bingeing on unhealthy food and purging after, to induce vomiting

A medical-surgical nurse works with a patient diagnosed with a somatic system disorder. Care planning is facilitated by understanding that the patient will probably: a. Attend group therapy sessions without encouragement. b. Readily seek psychiatric help c. Readily seek psychiatric counseling. d. Be resistant to accepting psychiatric help.

d. Be resistant to accepting psychiatric help.

The nurse provides care for clients in the pediatric clinic. The nurse understands that according to Erikson Stages of psychosocial development, trust and significant early attachments develop during which time period? a. 6 to 12 years b. 12 to 19 years c. 1 to 3 years d. Birth to 18 months

d. Birth to 18 months

Which is associated with bulimia nervosa? a. Fluid and electrolyte overload b. Hypertrophic and parotid glands c. Enlarged parotid glands d. Calluses on the hands and fingers (Russell's sign)

d. Calluses on the hands and fingers (Russell's sign)

A patient being admitted to the eating disorders unit has a yellow cast to the skin and fine, downy hair covering the body. The patient weighs 70 pounds; height is 5 feet, 4 inches. The patient is quiet and says only, "I won't eat until I look thin." What is the priority initial nursing diagnosis? a. Disturbed body image, related to too much weight loss b. Ineffective coping, related to lack of conflict resolution skills c. Anxiety, related to fear of gain a lot of weight d. Imbalanced nutrition: less than body requirements, related to self-starvation

d. Imbalanced nutrition: less than body requirements, related to self-starvation

The registered nurse observes that a female client with delusional disorder carries some of her belonging with her because she believes the others are trying to steal them. Which nursing action will promote trust? a. Offer to keep the belongings at the nurse deck b. Explain that distrust is related with feeling anxious c. Explain that these believes are related to her illness d. Initiate short, frequent contacts with the client

d. Initiate short, frequent contacts with the client

When preparing a teaching plan for a client who is to be discharged with a prescription for buspirone (Buspar), it is most important for the nurse to include which instruction? a. Avoid eating aged cheese and chicken liver. b. Eat foods high in fiber such as whole grain breads every day c. Always eat a same amount of sodium (salt0 d. It may take 3 to 4 weeks to achieve therapeutic effects

d. It may take 3 to 4 weeks to achieve therapeutic effects

Patient tells the nurse that he wants to kill his boss, the registered nurse tells healthcare provider, healthcare provider tells his boss. What disciplinary action is needed? a. File an incident report because violation of privacy. b. Both, the nurse and the healthcare provided did violation of confidentiality c. Inform the Board of Nursing that violation of patient's privacy d. None. The action was appropriate due to duty to warm law

d. None. The action was appropriate due to duty to warm law

During the admission of a male client to the mental health unit, the client tells the nurse that he had a panic attack today and ran out of the physicians office. Which questions is most important for the registered nurse to ask this client? a. Have you had any thoughts of hurting yourself? b. On a scale of 1 t 10 how do you rate your anxiety level? c. How would you describe your mood right now? d. What medications have you taken in the last 24 hours?

d. What medications have you taken in the last 24 hours?

A client is to begin lithium carbonate therapy. The nurse will ensure that the client has completed what baseline lab work before the drug administration? a. Blood sugar b. CBC c. liver studies d. creatinine and BUN

d. creatinine and BUN

A registered nurse cares for an older adult patient admitted for the treatment of depression. The health care provider prescribes an anti-anxiety medication today, but the dose is more than the usual adult dose. The nurse should: a. give the usual geriatric dosage as per guidelines b. consult the health care provider. c. consult a drug reference manual before administer the medication d. hold the medication and consult the health care provider

d. hold the medication and consult the health care provider

Which individual with a mental illness may need emergency involuntary hospitalization for mental illness? The individual who: a. resumes using heroin while still taking treatment with methadone. b. does not show up for an outpatient appointment with the mental health nurse. c. reports hearing angels playing harps during thunderstorms. d. refuses eating during last 2 weeks to weight lost and patient's weight is 80 pounds , blood pressure is 66/40 mm Hg, and heart rate is 38 per minute

d. refuses eating during last 2 weeks to weight lost and patient's weight is 80 pounds , blood pressure is 66/40 mm Hg, and heart rate is 38 per minute

A patient should be considered for involuntary hospitalization for psychiatric care when he or she: a. is noncompliant with the treatment regimen ordered by Psychiatric doctor. b. fraudulently files for bankruptcy twice. c. sells and distributes illegal drugs in the community every day. d. threatens to harm self.

d. threatens to harm self.

Which individual with a mental illness may need emergency involuntary hospitalization for mental illness? The individual who: a. does not show up for a very important appointment in the Mental health clinic b. resumes using heroine every day while still taking treatment with methadone c. reports hearing angels playing harps during storms d. want to hurt a neighbor due to command hallucinations

d. want to hurt a neighbor due to command hallucinations


Conjuntos de estudio relacionados

Adaptive Quiz: Basic Pharmacology for Nurses Ch. 9

View Set

Interaction Between People and Environment - Flocabulary

View Set

Practice Quiz 7.1 (RHIA & RHIT)Practice Quiz 7.2 (RHIA & RHIT) Practice Quiz 7.3 (RHIA & RHIT)CPT Coding Practice Quiz 4 (RHIA/RHIT)JUST CPT CODING

View Set

Physics 101 (geometry) Midterm (ch 1-7)

View Set

Topic 14 Lesson 5 (England) and Lesson 8 (Eastern Europe and Russia)

View Set

congress self-assessment questions - unit 2a review

View Set